Google Groups no longer supports new Usenet posts or subscriptions. Historical content remains viewable.
Dismiss

Why isn't the mathematician Henri Poincaré acknowledged as the true discoverer of special relativity?

28 views
Skip to first unread message

Perspicacious

unread,
Aug 3, 2005, 8:56:49 PM8/3/05
to
Why isn't the mathematician Henri Poincaré
acknowledged as the true discoverer of the
special theory of relativity?

http://www-cosmosaf.iap.fr/Poincare-RR3A.htm
http://arxiv.org/abs/physics/0408077


Harry

unread,
Aug 4, 2005, 11:11:12 AM8/4/05
to

"Perspicacious" <iperspi...@yahoo.com> wrote in message
news:1123101871.4...@g14g2000cwa.googlegroups.com...

Poincare was more than a mathematician, physics was an important hobby of
him.

Lorentz, Poincare and Einstein are all credited in good text books for their
contributions:

- Lorentz to first propose a rudimentory version of SRT
- Poincare to next come up with the Lorentz Transformations in order to
fulfill the PoR
- Einstein to next derive the LT from the PoR

And there were (of course) also some others who are worth of credit.

Often for simplicity only Einstein is mentioned because he developed the
general theory of relativity as well.

Harald


cma...@yahoo.com

unread,
Aug 4, 2005, 11:11:12 AM8/4/05
to

The answer to your question is partly "Eddington". By that, I mean that
Einstein became a superstar to the general public, whereas this never
happened to Poincare. When the war was over in 1918, Eddington
generated a lot of hype around Einstein's relativity, which was largely
unknown at the time. In this post-war atmosphere he claimed that there
were theological consequences to this theory, much to Einstein's
consternation. Anyway, Eddington was probably acting with
self-interests in mind, as he was already planning to test the light
bending prediction. Once this was verified in 1919, Einstein's fame was
catapulted to super-stardom. Well guess what, people then learned about
SR, and do you think Poincare really had to be mentioned in the media?

That being said, I can buy Einstein's claim that he was unaware of
Poincare's work; but it's hard to imagine that the editor of Annalen
der Physik wasn't: this person was none other than Max Planck.

Chris


Martin Ouwehand

unread,
Aug 4, 2005, 11:11:13 AM8/4/05
to
In the article <1123101871.4...@g14g2000cwa.googlegroups.com>,
Perspicacious <iperspi...@yahoo.com> writes:

] Why isn't the mathematician Henri Poincaré acknowledged as the true


] discoverer of the special theory of relativity?

because he didn't discover the theory of relativity as we now understand it.

It is true that after all the failed attempts to show the movement of the
earth with respect to the aether, he and Lorentz did realise that the
principle of relativity must be true and that it is impossible to detect
a translation movement with constant velocity.

But despite this, Poincaré still believed somehow in the aether and that
the speed of light is *not* the same in all inertial frames: for example
in his 1909 Conference [1], he explains that it is not possible to
synchronize two clocks A and B *which are mutually at rest* by exchanging
light signals because, if they were moving (I guess with respect to the
aether), the time for trip A->B wouldn't be the same as the time for trip
B->A, by an amount which it is impossible to determine, because the
principle of relativity.

He certainly did not explain, as Einstein did, the changes to the concepts
of space and time that follow from the theory of relativity.

All claims about Poincaré's priority come from the fact that in his
article [2] "Sur la Dynamique de l'Électron" he does show that the Maxwell
equations are invariant under the Lorentz transformation. But this article
is a follow-up to an article by Lorentz [3] were the Lorentz transformation
is presented *as a formal change of variables* used to show that the Lorentz
contraction, together with an hypothesis about the transformation of forces,
would explain why it is impossible to detect any movement with respect to
the aether. Indeed, Poincaré sometimes uses a phrase like "the real electron
corresponds through the Lorentz transformation to an ideal electron..."
Nowhere do Lorentz or Poincaré say that the Lorentz transformation connects
space-time measurements in two inertial frames in relative motion.

Finally, the bulk of Poincaré's article is devoted to a *dynamical*
explanation of the Lorentz contraction (he has to postulate some kind of
"pressure" inside the electron to do this), whereas we now believe with
Einstein that the Lorentz contraction is purely kinematical.

References:

[1] H. Poincaré [1909], Conférence devant l'Association française pour
l'Avancement des Sciences
[2] H. Poincaré [1906], "Sur la Dynamique de l'Électron", Rend. Circ. Mat.
Palermo XXI, p. 129
[3] H. A. Lorentz [1904], "Electromagnetic phenomena in systems moving with
any velocity less than that of light", Proc. Amst. Acad. VI, p. 809

[1] and [2] are available online in "La Mécanique Nouvelle":

http://gallica.bnf.fr/document?O=N029067

[3] is available in Dover's "The Principles of Relativity":

http://store.yahoo.com/doverpublications/0486600815.html

--
| ~~~~~~~~ Martin Ouwehand ~ Swiss Federal Institute of Technology ~ Lausanne
__|_____________ Email/PGP: http://slwww.epfl.ch/info/Martin.html _____________
Spin is an aspect of quantum mechanics [John Markoff in]
that is still untapped commercially [the Herald Tribune]

Nick Maclaren

unread,
Aug 4, 2005, 4:27:03 PM8/4/05
to

In article <42f201e3$1...@epflnews.epfl.ch>,

Harry <harald.v...@epfl.ch> writes:
|>
|> Lorentz, Poincare and Einstein are all credited in good text books for their
|> contributions:
|>
|> - Lorentz to first propose a rudimentory version of SRT
|> - Poincare to next come up with the Lorentz Transformations in order to
|> fulfill the PoR
|> - Einstein to next derive the LT from the PoR
|>
|> And there were (of course) also some others who are worth of credit.

Yes.

This is the basis of my objection to the claim that special
relativity would not have been developed if Einstein had not
existed. As with Newton's laws (which Hooke claimed to have
discovered, with some justification), there were a lot of people
converging on the solution. Special relativity would have been
proposed within a decade or so, anyway.

Whether that is true of general relativity is less clear.


Regards,
Nick Maclaren.

harald.v...@epfl.ch

unread,
Aug 4, 2005, 11:04:49 PM8/4/05
to

In my first message Poincare appears rather unimportant, but you now
remind me that I left out to mention that the modern PoR was,
apparently, first proposed by Poincare; and that we know from A. Pais
that a friend of Einstein witnessed to the fact that Einstein discussed
in great length a most important paper of Poincare - the effects of
which can be seen in Einstein's 1905 paper.

Harald


harald.v...@epfl.ch

unread,
Aug 4, 2005, 11:04:50 PM8/4/05
to
Some remarks and significant corrections:

Martin Ouwehand wrote:
> In the article <1123101871.4...@g14g2000cwa.googlegroups.com>,
> Perspicacious <iperspi...@yahoo.com> writes:
>
> ] Why isn't the mathematician Henri Poincaré acknowledged as the true
> ] discoverer of the special theory of relativity?
>
> because he didn't discover the theory of relativity as we now understand it.

The understanding of the theory lies outside of physics proper - such
interpretations can't be scientifically proven, nor have the
interpretations of Lorentz and Poincare been disproved - quite to the
contrary, see Builder, Aus.J.Ph., 1958, "Ether and Relativity".

> It is true that after all the failed attempts to show the movement of the
> earth with respect to the aether, he and Lorentz did realise that the
> principle of relativity must be true and that it is impossible to detect
> a translation movement with constant velocity.

If I understood well, this only became fully clear to Lorentz after
contemplating on Einstein's 1905 paper. Poincare's "note" of 1905 he
seems to have overlooked at that time, as you still do here (and it's
even on the web site of which you gave the link; BTW, thanks, it's
handy!)

> But despite this, Poincaré still believed somehow in the aether and that
> the speed of light is *not* the same in all inertial frames: for example
> in his 1909 Conference [1], he explains that it is not possible to
> synchronize two clocks A and B *which are mutually at rest* by exchanging
> light signals because, if they were moving (I guess with respect to the
> aether), the time for trip A->B wouldn't be the same as the time for trip
> B->A, by an amount which it is impossible to determine, because the
> principle of relativity.

I think to have found that passage ("A in Paris, B in Berlin" -
right?). The way I read it, he showed to understand it perfectly well.
Indeed, Einstein explicitly emphasised the same point in his 1905 paper
(surely because he based himself on earlier writings of Poincare):
light speed is defined as two-way average speed, and the two one-way
times are taken to be equal *by convention*, or, as he formulated it,
"by definition":
"We have not defined a common ``time'' for A and B, for the latter
cannot be defined at all unless we establish by definition that the
``time'' required by light to travel from A to B equals the ``time'' it
requires to travel from B to A"

> He certainly did not explain, as Einstein did, the changes to the concepts
> of space and time that follow from the theory of relativity.
> All claims about Poincaré's priority come from the fact that in his
> article [2] "Sur la Dynamique de l'Électron"

Not so: they relate to his shorter 1905 note with the same title (in
which he presented the Lorentz transformations and declared that they
obeyed the PoR), as well as to his earlier papers that inspired both
Lorentz and Einstein and lead the way to relativity.

> he does show that the Maxwell
> equations are invariant under the Lorentz transformation. But this article
> is a follow-up to an article by Lorentz [3] were the Lorentz transformation
> is presented *as a formal change of variables* used to show that the Lorentz
> contraction, together with an hypothesis about the transformation of forces,
> would explain why it is impossible to detect any movement with respect to
> the aether. Indeed, Poincaré sometimes uses a phrase like "the real electron
> corresponds through the Lorentz transformation to an ideal electron..."
> Nowhere do Lorentz or Poincaré say that the Lorentz transformation connects
> space-time measurements in two inertial frames in relative motion.

I am flabbergasted! A child can understand that the Michelson-Morley
experiment on motion through space measures exactly that, and both
Poincare and Lorentz explicitly set out to explain why measurements in
such frames gave a null result for the ether theory of that time. Did
you ever read the Michelson-Morley paper?

However, Lorentz later admitted that by 1905 he had not yet understood
the implications of his "local time", and I think that Einstein was the
first to give an example calculation of time dilation.

> Finally, the bulk of Poincaré's article is devoted to a *dynamical*
> explanation of the Lorentz contraction (he has to postulate some kind of
> "pressure" inside the electron to do this), whereas we now believe with
> Einstein that the Lorentz contraction is purely kinematical.

You believe so, but I don't: the "kinematical" explanation is IMO no
explanation at all - on that point I agree with Newton, Lorentz and
Poincare.
Personal belief *should* have no place in discussions of modern physics
, except to explain one's bias.
Nevertheless that popular bias may be a valid additional cause for the
relative silence about both Lorentz and Poincare.

NOte: after verifying from the original papers, I found that many text
books provide misinformation when it comes to the history of science
and the discussion of the merits of different theories/philosophies - I
feel like an idiot to have trusted such books, on that point "science"
does no better than religious sects. But that is worth a separate
thread. ;-)

Cheers,
Harald

Martin Ouwehand

unread,
Aug 5, 2005, 6:45:02 AM8/5/05
to
In article <1123192475.3...@g44g2000cwa.googlegroups.com>,
harald.v...@epfl.ch writes:

] Martin Ouwehand wrote:
]
] > It is true that after all the failed attempts to show the movement of the


] > earth with respect to the aether, he and Lorentz did realise that the
] > principle of relativity must be true and that it is impossible to detect
] > a translation movement with constant velocity.
]
] If I understood well, this only became fully clear to Lorentz after
] contemplating on Einstein's 1905 paper.

Lorentz writes in his 1904 paper: "It would be more satisfactory if it
were possible to show [...] that many electromagnetic actions are entirely
independent of the motion of the system." Taking in account the careful
and prudent style of Lorentz, I understand by this that he realised that
the principle of relativity must be true.

] Poincare's "note" of 1905 he
] seems to have overlooked at that time, as you still do here.

well, it's obviously just a short summary of his 1906 paper read before
the Académie des Sciences.

] > But despite this, Poincaré still believed somehow in the aether and that


] > the speed of light is *not* the same in all inertial frames: for example
] > in his 1909 Conference [1], he explains that it is not possible to
] > synchronize two clocks A and B *which are mutually at rest* by exchanging
] > light signals because, if they were moving (I guess with respect to the
] > aether), the time for trip A->B wouldn't be the same as the time for trip
] > B->A, by an amount which it is impossible to determine, because the
] > principle of relativity.
]
] I think to have found that passage ("A in Paris, B in Berlin" -
] right?). The way I read it, he showed to understand it perfectly well.
] Indeed, Einstein explicitly emphasised the same point in his 1905 paper
] (surely because he based himself on earlier writings of Poincare):

oh no, for Einstein it *is* possible to synchronize two clock at rest
with respect to each other in an inertial frame, because he postulates
that the speed of light is the same in every inertial frame. And that's
the point: whatever Poincaré's theory was (and from this conference for
lay persons I find it difficult to understand his theory of space and
time -- I don't know if he wrote a "professional" paper where he explains
it better), it's not relativity theory as we now undersand it.

] > Nowhere do Lorentz or Poincaré say that the Lorentz transformation


] > connects space-time measurements in two inertial frames in relative
] > motion.
]
] I am flabbergasted!

let's follow Lorentz' reasoning in his 1904 paper: he writes down
the Maxwell equation in the aether frame (x, t), he then writes
them down in a frame (x', t') moving with speed v with respect to
the aether, using a *galilean* transformation:

x' = x - v * t
t' = t

(then the Maxwell equations aren't the same as in the aether frame) and
then he performs a transformation on this sytem in motion, saying "I take
(x'', t'') as new independent variables", to show that the transformed
system behaves enough like a system at rest with respect to the aether
that it's possible to deduce, by doing the inverse transformation, that
the translation motion v isn't detectable. Mathematically, the transformation
(x, t) -> (x'', t'') is what we call a Lorentz transformation, but it
seems clear to me that for Lorentz the transformation of space-time
measurement between the two frames is (x, t) -> (x', t'), not (x, t) ->
(x'' ,t''). This is what I meant by the above comment "Nowhere do they..."
Poincaré's 1906 paper is silent about the space-time content of the Lorentz
transformation, but as it's a follow-up to Lorentz' I take it that he
agrees with him on this point.

--
| ~~~~~~~~ Martin Ouwehand ~ Swiss Federal Institute of Technology ~ Lausanne
__|_____________ Email/PGP: http://slwww.epfl.ch/info/Martin.html _____________

It is a brave and foolhardy and desperate
man who will perform an autopeotomy [Simon Winchester]

cma...@yahoo.com

unread,
Aug 5, 2005, 6:45:01 AM8/5/05
to

It's hard for me to doubt the sincerity of such a great man, but yes
there are troubling facts that are hard to swallow. I found
Perspicacious' links very interesting, especially the arxiv document. I
have merely glanced at it yet, and already I keep telling myself "say
it ain't so". Just consider the fact that it is Poincare who gave the
equations that he dubbed himself the "Lorentz transformations", and
there you have Einstein using exactly the same term, and decades later
he claimed that he only was aware of Lorentz's work in 1895. Pure
coincidence? Maybe, but as I said: hard to swallow. So far I've barely
glanced at Poincare's discussion on silmulataneity, but it was enough
to give me a feeling of deja vu.

What's my take on all this? I think Einstein was mostly interested in
discussing electrodynamics. He probably thought that the SR part of it
was already commonly accepted. So he simply summarized what was already
known albeit very elegantly, so to be able to move on to his main
subject "electrodynamics". Later on in 1918, 1919, seeing that he was
getting all the credit for this as well, he simply kept his mouth shut.

Chris


Martin Ouwehand

unread,
Aug 7, 2005, 2:25:20 PM8/7/05
to
In article <1123217831....@o13g2000cwo.googlegroups.com>,
cma...@yahoo.com writes:

] Just consider the fact that it is Poincare who gave the


] equations that he dubbed himself the "Lorentz transformations", and
] there you have Einstein using exactly the same term

no, the term "Lorentz transformation" doesn't appear in Einstein's
1905 paper -- BTW it's available from:

http://www3.interscience.wiley.com/cgi-bin/jissue/109924399

] I think Einstein was mostly interested in discussing electrodynamics.

well at least his 1905 paper has chapters called "Definition of
Simultaneity", "On the Relativity of Lengths and Times", etc. Similar
explanations are what I find sorely missing in Poincaré's 1906 paper to
qualify as an outline of relativity theory. On the other hand, it
extensively discusses an electrodynamical model of the electron.

--
| ~~~~~~~~ Martin Ouwehand ~ Swiss Federal Institute of Technology ~ Lausanne
__|_____________ Email/PGP: http://slwww.epfl.ch/info/Martin.html _____________

Science is the belief in the ignorance of experts [Richard P. Feynman]

Harry

unread,
Aug 7, 2005, 2:25:19 PM8/7/05
to

"Martin Ouwehand" <see...@end.of.post.ch> wrote in message
news:42f33703$1...@epflnews.epfl.ch...

> In article <1123192475.3...@g44g2000cwa.googlegroups.com>,
> harald.v...@epfl.ch writes:
>
> ] Martin Ouwehand wrote:
> ]
> ] > It is true that after all the failed attempts to show the movement of
the
> ] > earth with respect to the aether, he and Lorentz did realise that the
> ] > principle of relativity must be true and that it is impossible to
detect
> ] > a translation movement with constant velocity.
> ]
> ] If I understood well, this only became fully clear to Lorentz after
> ] contemplating on Einstein's 1905 paper.
>
> Lorentz writes in his 1904 paper: "It would be more satisfactory if it
> were possible to show [...] that many electromagnetic actions are entirely
> independent of the motion of the system." Taking in account the careful
> and prudent style of Lorentz, I understand by this that he realised that
> the principle of relativity must be true.

OK - still he admitted himself later that the consequences of his theory had
not been fully clear to him.

> ] Poincare's "note" of 1905 he
> ] seems to have overlooked at that time, as you still do here.
>
> well, it's obviously just a short summary of his 1906 paper read before
> the Académie des Sciences.

I agree, but as 1906 is later than Einstein's paper, for the priority it's
important to notice it.

> ] > But despite this, Poincaré still believed somehow in the aether and
that
> ] > the speed of light is *not* the same in all inertial frames: for
example
> ] > in his 1909 Conference [1], he explains that it is not possible to
> ] > synchronize two clocks A and B *which are mutually at rest* by
exchanging
> ] > light signals because, if they were moving (I guess with respect to
the
> ] > aether), the time for trip A->B wouldn't be the same as the time for
trip
> ] > B->A, by an amount which it is impossible to determine, because the
> ] > principle of relativity.
> ]
> ] I think to have found that passage ("A in Paris, B in Berlin" -
> ] right?). The way I read it, he showed to understand it perfectly well.
> ] Indeed, Einstein explicitly emphasised the same point in his 1905 paper
> ] (surely because he based himself on earlier writings of Poincare):
>
> oh no, for Einstein it *is* possible to synchronize two clock at rest
> with respect to each other in an inertial frame, because he postulates
> that the speed of light is the same in every inertial frame.

No, see immediately below in my posting: the "speed of light" is defined as
being two-way, while the one-way speed is purely a matter of convention.
(Did you overlook to correct this before sending your message or didn't you
understand this point? But I now see that in your reply you snipped that
part.. maybe you forgot to read it?)

> And that's
> the point: whatever Poincaré's theory was (and from this conference for
> lay persons I find it difficult to understand his theory of space and
> time -- I don't know if he wrote a "professional" paper where he explains
> it better), it's not relativity theory as we now undersand it.

It's perfectly clear to me and exactly like I understand relativity theory.
The *measurable* speed of light was for Poincare the same in all inertial
frames, as concluded from M&M and so on. He just described things from a
different perspective than Einstein, as his philosophy was similar to
Lorentz.
Everyone who knows SRT knows that synchronization is "relative" ... I even
read somewhere that Einstein's synchronisation convention actually first was
proposed by Poincare!

> ] > Nowhere do Lorentz or Poincaré say that the Lorentz transformation
> ] > connects space-time measurements in two inertial frames in relative
> ] > motion.
> ]
> ] I am flabbergasted!
>
> let's follow Lorentz' reasoning in his 1904 paper:

OK, Lorentz introduced his paper with (emphasis mine):

"The problem of determining the influence exerted on
*electric and optical phenomena by a translation*,
*such as all systems have in virtue of the Earth's annual motion*,
admits of a comparatively simple solution, so long
as only those terms need be taken into account, which are proportional to
the first power of the ratio between the velocity of translation v and
the velocity of light c. Cases in which quantities of the second order,
i.e. of the order (v/c)^2, may be perceptible, present more difficulties.
The first example of this kind is Michelson's well-known interference
experiment, the negative result of which has led Fitzgerald and myself to
the conclusion that the dimensions of solid bodies are slightly altered
by their motion through the ether."

> he writes down
> the Maxwell equation in the aether frame (x, t), he then writes
> them down in a frame (x', t') moving with speed v with respect to
> the aether, using a *galilean* transformation:
>
> x' = x - v * t
> t' = t

Einstein does something similar in his 1905 paper, simply replacing "ether"
by "rest". I'm afraid that I miss your point...

> (then the Maxwell equations aren't the same as in the aether frame) and
> then he performs a transformation on this sytem in motion, saying "I take
> (x'', t'') as new independent variables", to show that the transformed
> system behaves enough like a system at rest with respect to the aether
> that it's possible to deduce, by doing the inverse transformation, that
> the translation motion v isn't detectable.

That's what matters, doesn't it?

> Mathematically, the transformation
> (x, t) -> (x'', t'') is what we call a Lorentz transformation, but it
> seems clear to me that for Lorentz the transformation of space-time
> measurement between the two frames is (x, t) -> (x', t'), not (x, t) ->
> (x'' ,t''). This is what I meant by the above comment "Nowhere do they..."
> Poincaré's 1906 paper is silent about the space-time content of the
Lorentz
> transformation, but as it's a follow-up to Lorentz' I take it that he
> agrees with him on this point.

I'm still not sure what you mean there: I don't see how one can misapply a
Lorentz transformation and still obtain the correct predictions!
OTOH, I noticed how Poincare was enthousiast about Lorentz' "local time"
concept, while Lorentz had not really thought much about it himself until
after he read Einstein's paper.

> | ~~~~~~~~ Martin Ouwehand ~ Swiss Federal Institute of Technology ~
Lausanne

Nice, we're in the same institute, I had not noticed that!
-> Let's go for a coffee break next week, maybe together with prof. Gruber
who is also interested in this subject. What do you say?

Harald


Phil

unread,
Aug 7, 2005, 2:25:49 PM8/7/05
to

"Perspicacious" <iperspi...@yahoo.com> wrote in message
news:1123101871.4...@g14g2000cwa.googlegroups.com...
> Why isn't the mathematician Henri Poincaré
> acknowledged as the true discoverer of the
> special theory of relativity?

There is a variety of reasons, some are socialogical and some
are linked to differences in the philiosphical approaches that
Poincaré and Einstein took.

Poincaré held to a philosphy of conventionalism. He thought that
even if there was no way to detect the frame of reference of the
ether, it is nonetheless a necessary concept and its frame should be
adopted by a convention. He was influenced by the practical
necessity to define international standards of time and measurement.

Eistein was more influenced by positivism and philiosophical
teachings on the relativity of knowledge. He realised that
conventions were only practical necessities for measuement and
were not what matters in fundamental physics so he banished the
ether altogether. Minkowski made it even clearer when he
introduced space-time.

The difference is philosophical but it is still very significant. Eisntein
was able to extend his ideas to general relativity by applying the
mathematics of curved space-time. Poincaré had been very aware
of non-Euclidean geometry but he believed that people would always
adopt a convention of measuring with Euclidean geometry because
it is simpler. With such a point of view general relativity would have
been very difficult to discover.

An instructive book about this aspect of the story is "Einstein's
Clocks. Poincaré's Maps" by Peter Galison.

Phillip Helbig---remove CLOTHES to reply

unread,
Aug 7, 2005, 6:04:44 PM8/7/05
to

> It's hard for me to doubt the sincerity of such a great man, but yes
> there are troubling facts that are hard to swallow. I found
> Perspicacious' links very interesting, especially the arxiv document. I
> have merely glanced at it yet, and already I keep telling myself "say
> it ain't so". Just consider the fact that it is Poincare who gave the
> equations that he dubbed himself the "Lorentz transformations", and
> there you have Einstein using exactly the same term, and decades later
> he claimed that he only was aware of Lorentz's work in 1895. Pure
> coincidence? Maybe, but as I said: hard to swallow. So far I've barely
> glanced at Poincare's discussion on silmulataneity, but it was enough
> to give me a feeling of deja vu.

I think it's important to distinguish between intentional
misrepresentation of what really happened, the memories of an old man
about the sequence of his thoughts many years before, and possibly an
intentional shift of emphasis to highlight what is really important (for
example, even if Einstein had been motivated by the null result of the
Michelson-Morley experiment, he could have deemed it worthwhile to
downplay this and concentrate on the development of SR from first
principles as opposed to an explanation for a "failed" experiment).
(Perhaps George Harrison really did appropriate the melody from "He's so
fine" for "My Sweet Lord". True, he had to pay half a million dollars
as a fine (peanuts for him, of course, but a warning to the less rich
and/or less talented not to try something similar), but even the court
said that any appropriation was unintentional, as it was clear that
George Harrison was perfectly capable of writing original melodies. The
situation with Einstein and Poincaré could be similar.)

Phillip Helbig---remove CLOTHES to reply

unread,
Aug 7, 2005, 6:04:44 PM8/7/05
to
In article <1123155333....@g43g2000cwa.googlegroups.com>,
cma...@yahoo.com writes:

> The answer to your question is partly "Eddington". By that, I mean that
> Einstein became a superstar to the general public, whereas this never
> happened to Poincare. When the war was over in 1918, Eddington
> generated a lot of hype around Einstein's relativity, which was largely
> unknown at the time. In this post-war atmosphere he claimed that there
> were theological consequences to this theory, much to Einstein's
> consternation.

He preferred his own "Eddington model" of the universe, which is
asymptotic to the Einstein static universe in the infinite past, to a
"big bang" since the beginning should be "not to aesthetically abrupt".
He was a Quaker who never married. When asked by a reporter if it were
true that only three people in the world understood General Relativity,
he paused and then said "I'm trying to think who the third person could
be". Strange character, great astrophysicist, very influential in
popularising Einstein, got lost in unified field theory (as did
Einstein), though in Eddington's case it's more like a comedy of errors
than the tragedy of Einstein's case.

> Anyway, Eddington was probably acting with
> self-interests in mind, as he was already planning to test the light
> bending prediction. Once this was verified in 1919, Einstein's fame was
> catapulted to super-stardom. Well guess what, people then learned about
> SR, and do you think Poincare really had to be mentioned in the media?
>
> That being said, I can buy Einstein's claim that he was unaware of
> Poincare's work; but it's hard to imagine that the editor of Annalen
> der Physik wasn't: this person was none other than Max Planck.

Sometimes its Poincaré, sometimes it's Mileva Maric, sometimes it's
Lorentz who was supposed to have been there first, only to get cheated
out by Einstein; sometimes it is the statement that OF COURSE Einstein
was heavily influenced by the Michelson-Morley experiment. All of these
claims have been debunked. See, for example, Pais's excellent SUBTLE IS
THE LORD. (The title is a translation of Einstein's famous phrase
"Raffiniert ist der Herrgott, aber boshaft ist er nicht; Einstein's own
translation was "God is slick, but he ain't mean".)

Perspicacious

unread,
Aug 7, 2005, 10:19:27 PM8/7/05
to
Phillip Helbig wrote:
> I think it's important to distinguish between intentional
> misrepresentation of what really happened ...

OK then. Let's contrast the intentional misrepresentations
of physics lore with actual historical facts. Here's a
confirmation of what Chris wrote previously:

http://groups-beta.google.com/group/sci.physics.research/msg/398700efdcfa3ccc
http://groups-beta.google.com/group/sci.physics.research/msg/af718d896054763f

Eddington's observations confirmed Einstein's
theory, and were hailed at the time as a
conclusive proof of general relativity over
the Newtonian model; the news was reported
in newspapers all over the world as a major
story. It is also the source of the urban
rumor that only three people understand
relativity; when asked by a reporter who
suggested this, Eddington jokingly replied
"Oh, who's the third?"

However, recent historical examinations of
the case have shown that the raw data was
inconclusive, and that Eddington was
arbitrarily selective in choosing which
results to use. For a detailed account,
see [the link] predictive power.
http://en.wikipedia.org/wiki/Arthur_Eddington

Harry

unread,
Aug 8, 2005, 10:59:33 AM8/8/05
to

"Phillip Helbig---remove CLOTHES to reply" <hel...@astro.multiCLOTHESvax.de>
wrote in message news:dd5n5t$ji9$5...@online.de...
SNIP

> > That being said, I can buy Einstein's claim that he was unaware of
> > Poincare's work; but it's hard to imagine that the editor of Annalen
> > der Physik wasn't: this person was none other than Max Planck.
>
> Sometimes its Poincaré, sometimes it's Mileva Maric, sometimes it's
> Lorentz who was supposed to have been there first,

Supposedly been where first? All papers are there for everyone to read...

> only to get cheated
> out by Einstein; sometimes it is the statement that OF COURSE Einstein
> was heavily influenced by the Michelson-Morley experiment. All of these
> claims have been debunked. See, for example, Pais's excellent SUBTLE IS
> THE LORD. (The title is a translation of Einstein's famous phrase
> "Raffiniert ist der Herrgott, aber boshaft ist er nicht; Einstein's own
> translation was "God is slick, but he ain't mean".)

You should read Pais' book again! See my reply higher up in this thread.

Harald


Phillip Helbig---remove CLOTHES to reply

unread,
Aug 8, 2005, 10:59:32 AM8/8/05
to
In article <1123461168.8...@f14g2000cwb.googlegroups.com>,
Perspicacious <iperspi...@yahoo.com> writes:

All true. However, I was referring to whether or not Einstein was
influenced by Poincaré etc, not to his fame as a result of Eddington's
PR, which I think no-one disputes. (The original observations were
inconclusive. Of course, since then Einstein's prediction has been
confirmed many times.)

Martin Ouwehand

unread,
Aug 8, 2005, 3:03:11 PM8/8/05
to
In article <42f3859a$1...@epflnews.epfl.ch>,
Harry <harald.v...@epfl.ch> writes:

] "Martin Ouwehand" <see...@end.of.post.ch> wrote in message
] news:42f33703$1...@epflnews.epfl.ch...
]
] > well, it's obviously just a short summary of his 1906 paper read before


] > the Académie des Sciences.
]
] I agree, but as 1906 is later than Einstein's paper, for the priority it's
] important to notice it.

the priority question would arise only if Poincaré's 1905 note, 1906 article,
or anything else he wrote, contained relativity theory or something equivalent
to it. I don't think it does.

] (Did you overlook to correct this before sending your message or didn't you


] understand this point? But I now see that in your reply you snipped that
] part.. maybe you forgot to read it?)

my point was that Poincaré's theory is not the same as Einstein's: in his
1905 article, Einstein explains how to synchronize clocks which are mutually
at rest, while in this 1909 Conférence Poincaré explains that it can't be
done ("il leur est absolument impossible de savoir si leurs chronomètres
marquent ou non la même heure") -- and from the context I understand that
it's because he believes that the speed of light in a moving frame is
not the same as its speed with respect to the aether (another difference
with Einstein.)

I think it answers in part the question that started this thread, but it
seems what you are saying is more something like "Poincaré's theory is
equivalent to Einstein's, if properly interpreted" -- but then I think it's
either not true, or you won't find references in Poincaré's writings to
back up your interpretation. Take for instance time dilatation: A tells
to B "please take this train riding at 80% of the speed of light, and
let this dynamite stick explode after 10 seconds", how would Poincaré
have determined the time of the explosion as seen by an observer next
to the rail at the moment of the explosion and at rest with respect to A ?
would his answer be correct ?

] > he writes down


] > the Maxwell equation in the aether frame (x, t), he then writes
] > them down in a frame (x', t') moving with speed v with respect to
] > the aether, using a *galilean* transformation:
] >
] > x' = x - v * t
] > t' = t
]
] Einstein does something similar in his 1905 paper, simply replacing "ether"
] by "rest".

No, Einstein ends up with:

x' = (x - v * t) / sqrt(1 - (v/c)^2)
t' = (t - v * x/c^2) / sqrt(1 - (v/c)^2)

You know, the Lorentz transformation :-)

] I'm afraid that I miss your point...

it's for those people who say "Look ! a Lorentz transformation ! it's
the theory of relativity !" -- for Lorentz it was some kind of mathematical
trick ("a change of variable"), not the transformation rule for space-time
measurements between two inertial frames, for which he still used the
galilean transformation.

] I'm still not sure what you mean there: I don't see how one can misapply a


] Lorentz transformation and still obtain the correct predictions!

some predictions of the resulting theory aren't correct: no time dilatation,
the time it takes for light to travel one way along a moving rod depends on
the direction (the Lorentz contraction cancels time differences only if light
travels up and down the moving rod.), the galilean transformation rule for
the electromagnetical field...

--

| ~~~~~~~~ Martin Ouwehand ~ Swiss Federal Institute of Technology ~ Lausanne

__|_____________ Email/PGP: http://slwww.epfl.ch/info/Martin.html _____________

So please, oh please, we beg, we pray,
Go throw your TV set away [Roald Dahl]

harald.v...@epfl.ch

unread,
Aug 8, 2005, 8:52:34 PM8/8/05
to

Martin Ouwehand wrote:
> In article <42f3859a$1...@epflnews.epfl.ch>,
> Harry <harald.v...@epfl.ch> writes:
>
> ] "Martin Ouwehand" <see...@end.of.post.ch> wrote in message
> ] news:42f33703$1...@epflnews.epfl.ch...
> ]
> ] > well, it's obviously just a short summary of his 1906 paper read before
> ] > the Académie des Sciences.
> ]
> ] I agree, but as 1906 is later than Einstein's paper, for the priority it's
> ] important to notice it.
>
> the priority question would arise only if Poincaré's 1905 note, 1906 article,
> or anything else he wrote, contained relativity theory or something equivalent
> to it. I don't think it does.

That was not the issue (which you snipped and next forgot). Never
mind...

> ] (Did you overlook to correct this before sending your message or didn't you
> ] understand this point? But I now see that in your reply you snipped that
> ] part.. maybe you forgot to read it?)
>
> my point was that Poincaré's theory is not the same as Einstein's: in his
> 1905 article, Einstein explains how to synchronize clocks which are mutually
> at rest, while in this 1909 Conférence Poincaré explains that it can't be
> done ("il leur est absolument impossible de savoir si leurs chronomètres
> marquent ou non la même heure") -- and from the context I understand that
> it's because he believes that the speed of light in a moving frame is
> not the same as its speed with respect to the aether (another difference
> with Einstein.)

I pointed out why your thinking is mistaken - in fact your
interpretation of the above citation shows a complete misunderstanding
by you, and Poincare had explained all these matters when Einstein was
just starting to get the hang of it. Please look again at what I
pointed out to you and reply that. Or just try (but I bet that's
impossible to do with your misunderstanding) to explain how Poincare
calculated with his LT ("which form a group") that the *measured*
"speed of light in a moving frame is not the same as its [true] speed
with respect to the aether".
The difference between Poincare and Einstein is that Poincare discussed
objective (but undetectable) reality, while for Einstein, reality is
what one observes (leading to multiple realities). That debate goes
back to the time of Plato (but is independent of experimental and
mathematical physics)!

> I think it answers in part the question that started this thread, but it
> seems what you are saying is more something like "Poincaré's theory is
> equivalent to Einstein's, if properly interpreted"

That's a rather well known fact...

> -- but then I think it's
> either not true, or you won't find references in Poincaré's writings to
> back up your interpretation.

My interpretation is based on journal articles and on studying the
original papers by Poincare and Einstein.

> Take for instance time dilatation: A tells
> to B "please take this train riding at 80% of the speed of light, and
> let this dynamite stick explode after 10 seconds", how would Poincaré
> have determined the time of the explosion as seen by an observer next
> to the rail at the moment of the explosion and at rest with respect to A ?
> would his answer be correct ?

Why not? - as long as "second" is unambiguously defined above. You seem
to be unaware that Poincare was the one who first prescribed the
"Einstein" synchronization method! (

> ] > he writes down
> ] > the Maxwell equation in the aether frame (x, t), he then writes
> ] > them down in a frame (x', t') moving with speed v with respect to
> ] > the aether, using a *galilean* transformation:
> ] >
> ] > x' = x - v * t
> ] > t' = t
> ]
> ] Einstein does something similar in his 1905 paper, simply replacing "ether"
> ] by "rest".
>
> No, Einstein ends up with:
>
> x' = (x - v * t) / sqrt(1 - (v/c)^2)
> t' = (t - v * x/c^2) / sqrt(1 - (v/c)^2)
>
> You know, the Lorentz transformation :-)

Indeed that's what Poincare first ended up with, and he named those the
Lorentz transformations. Now what? ;-)

> ] I'm afraid that I miss your point...
>
> it's for those people who say "Look ! a Lorentz transformation ! it's
> the theory of relativity !" -- for Lorentz it was some kind of mathematical
> trick ("a change of variable"), not the transformation rule for space-time
> measurements between two inertial frames, for which he still used the
> galilean transformation.

You seem to be mixing up things: Length contraction was not a
"mathematical trick" for Lorentz (and neither for Einstein).
Inconsistently, he didn't grasp yet that time dilation consequently
should be as real.
But based on his theory, Lorentz provided the first true prediction of
SRT, as verified in ca. 1909. Next Einstein gave the same prediction.
How can one do a prediction with a "mere change of variables"?! Please
stay away from empty insinuations.

> ] I'm still not sure what you mean there: I don't see how one can misapply a
> ] Lorentz transformation and still obtain the correct predictions!
>
> some predictions of the resulting theory aren't correct: no time dilatation,

Which theory and of whom? I am unable to apply the LT without obtaining
time dilation - as long as of course the symbols have the original
meaning. You will impress anyone here if you can do that!

> the time it takes for light to travel one way along a moving rod depends on
> the direction (the Lorentz contraction cancels time differences only if light
> travels up and down the moving rod.)

That is a fact as measured in the "stationary" frame... otherwise GPS
and Sagnac gyroscopes would malfunction!

, the galilean transformation rule for the electromagnetical field...

I don't know what you mean with that one - but seen the above, I don't
hold my breath...
-> Really, we next better discuss eye to eye with a cup of coffee
instead of on this NG!

Cheers,
Harald, IMS, EPFL.


Ilja Schmelzer

unread,
Aug 9, 2005, 4:06:01 AM8/9/05
to
"Martin Ouwehand" <see...@end.of.post.ch> schrieb

> Perspicacious <iperspi...@yahoo.com> writes:
>
> ] Why isn't the mathematician Henri Poincaré acknowledged as the true
> ] discoverer of the special theory of relativity?

> But despite this, Poincaré still believed somehow in the aether and that


> the speed of light is *not* the same in all inertial frames: for example
> in his 1909 Conference [1], he explains that it is not possible to
> synchronize two clocks A and B *which are mutually at rest* by exchanging
> light signals because, if they were moving (I guess with respect to the
> aether), the time for trip A->B wouldn't be the same as the time for trip
> B->A, by an amount which it is impossible to determine, because the
> principle of relativity.
>
> He certainly did not explain, as Einstein did, the changes to the concepts
> of space and time that follow from the theory of relativity.

Hm, the question is if they follow from the theory.

An ether interpretation of relativity with classical space and time but
distorted measurements is certainly in agreement with the facts and
the mathematical apparatus of special relativity.

The spacetime interpretation is only one of two interpretations,
it is not a logical consequence.

Ilja

Charles Francis

unread,
Aug 9, 2005, 10:40:16 AM8/9/05
to
In message <dd9fqg$4uk$1...@beech.fernuni-hagen.de>, Ilja Schmelzer
<Ilja.Sc...@FernUni-Hagen.de> writes

I think that perhaps explains why Einstein was indeed the discoverer of
special relativity. Relativity refers to relativity of motion, i.e. the
absence of an ether interpretation, not to the mathematical
transformations which were already known.


Regards

--
Charles Francis

Harry

unread,
Aug 10, 2005, 10:39:56 AM8/10/05
to

"Charles Francis" <cha...@clef.demon.co.uk> wrote in message
news:buyVt7Fk...@clef.demon.co.uk...

Metaphysical interpretation isn't part of modern physics (of which SRT
belongs), except maybe if it can be disproved by logic. But I agree that it
explains why so many *claim* that Einstein was the sole discoverer of SRT,
for at that time he suggested to have no need for an ether and that is the
popular philosophy (or should I say mantra or mime?) of today.

Harald


Charles Francis

unread,
Aug 11, 2005, 6:32:36 AM8/11/05
to
In message <42f9d355$1...@epflnews.epfl.ch>, Harry
<harald.v...@epfl.ch> writes

>>
>> I think that perhaps explains why Einstein was indeed the discoverer of
>> special relativity. Relativity refers to relativity of motion, i.e. the
>> absence of an ether interpretation, not to the mathematical
>> transformations which were already known.
>
>Metaphysical interpretation isn't part of modern physics (of which SRT
>belongs),

And ether does not.

>except maybe if it can be disproved by logic. But I agree that it
>explains why so many *claim* that Einstein was the sole discoverer of SRT,
>for at that time he suggested to have no need for an ether and that is the
>popular philosophy (or should I say mantra or mime?) of today.
>

He didn't suggest, he demonstrated.

Regards

--
Charles Francis

Levin

unread,
Aug 11, 2005, 2:06:40 PM8/11/05
to
In <42f9d355$1...@epflnews.epfl.ch>, Harry <harald.v...@epfl.ch> wrote:
: Metaphysical interpretation isn't part of modern physics

: (of which SRT belongs), except maybe if it can be disproved
: by logic. But I agree that it explains why so many *claim*
: that Einstein was the sole discoverer of SRT, for at that time
: he suggested to have no need for an ether and that is the popular
: philosophy (or should I say mantra or mime?) of today. Harald

Could you please explain how would it follow from Poincare or
Lorentz ideas that, say, radioactive decay works the same in all
inertial frames (not even talking about things like gravity).

Harry

unread,
Aug 12, 2005, 4:37:11 AM8/12/05
to
"Charles Francis" <cha...@clef.demon.co.uk> wrote in message
news:J9qW$mXyjl...@clef.demon.co.uk...

> In message <42f9d355$1...@epflnews.epfl.ch>, Harry
> <harald.v...@epfl.ch> writes
> >>
> >> I think that perhaps explains why Einstein was indeed the discoverer of
> >> special relativity. Relativity refers to relativity of motion, i.e. the
> >> absence of an ether interpretation, not to the mathematical
> >> transformations which were already known.
> >
> >Metaphysical interpretation isn't part of modern physics (of which SRT
> >belongs),
>
> And ether does not.

Sure - except as a model of course. Models can be used and replaced at will.
The "Space-Time" model is quite popular nowadays, perhaps because the model
is identical to the equations themselves on which it is based, allowing for
intellectual simplicity.

> >except maybe if it can be disproved by logic. But I agree that it
> >explains why so many *claim* that Einstein was the sole discoverer of
SRT,
> >for at that time he suggested to have no need for an ether and that is
the
> >popular philosophy (or should I say mantra or mime?) of today.
> >
> He didn't suggest, he demonstrated.

That's a bit subtle:
- He demonstrated to have no need to use any ether model as long as he
postulated one outcome of ether models that he accepted: that light
speed is independent of the motion of the source. Poincare made a
similar derivation (according to some, more elegantly), as well as
Lorentz later in his courses.
- Einstein admitted later that of course one needs some kind of ether
concept - or alternative labels such as "Space" or "field" (or,
nowadays, "vacuum") to explain a number of things in physics. But
apparently that's incompatible with the mantra that "There Is No Ether"
(repeat once a day).

Harald

Tim

unread,
Aug 12, 2005, 4:38:28 AM8/12/05
to
Didn't Hamilton propose a gravity theory with humps and dips in a
landscape some fifty years before 1916?

Tim

Paul Stowe

unread,
Aug 12, 2005, 4:39:11 AM8/12/05
to

> all inertial frames ...

Because, in the PLR (Poincare-Lorentz Relativity) interpretation
it is the medium's (aether) properties that is the root cause of
the observed effects. This in turn is because all elements of the
medium (including all forms of matter & energy) can only communicate
with each other via transference by the medium, at its
characteristic speed c. This speed is regulated by the local bulk
properties. Processes simply cannot occur any faster or slower,
period. Thus wavicle patterns which are in relative motion must
adjust, just to keep at an equilibrium state.

As for the weak interaction (radioactive decay), isn't that a
statistical process? Does not the probability of interaction
change if the surrounding energy pattern is changed? But these
details aren't necessary because the very same argument used in
SR is just as valid for PLR...

> ... (not even talking about things like gravity).

That's great since gravity isn't inertial by definition. But
the very same argument is applicable to this too.

Paul Stowe

Harry

unread,
Aug 12, 2005, 11:33:47 AM8/12/05
to

"Levin" <l...@csb.bu.edu> wrote in message news:ddfqd9$8vh$1...@news3.bu.edu...

I think that Paul Stowe tried to model it somehow, but maybe that is not
really what you asked.
Roughly (from memory):

- From Poincare we have the PoR: All laws of nature (incl. Maxwell's) are
independent of our state of inertial motion.
- From Lorentz+Poincare we have the LT in which t and t' necessarily stand
for time counters of any natural process in order to obey the PoR.

Thus from their ideas, radioactive decay must be a process (in the ether)
that is equally affected by (absolute) speed as any other process.

Cheers,
Harald


cma...@yahoo.com

unread,
Aug 12, 2005, 11:01:21 PM8/12/05
to

I don't know much about the Lorentz-Poincare theory, but perhaps this
could add another element to your argument.

It's my understanding that although he assumes an etherframe, Lorentz
showed that inertial frames are not distinguishable from the
etherframe(s). And consequently, you may conclude that all inertial
frames are etherframes. And therefore the Maxwell laws hold good in all
inertial frames, provided they hold in at least one of them.

In this theory the "etherframe" turns out to be a very vacuous concept;
it's only a frame wherein the Maxwell laws apply. Viewing it this way,
Einstein didn't bring anything new.

As I said, I don't know too much about this. So... trust but verify.

Chris

Levin

unread,
Aug 13, 2005, 2:21:52 AM8/13/05
to
In <42f9d355$1...@epflnews.epfl.ch>, Harry <harald.v...@epfl.ch> wrote:
: Metaphysical interpretation isn't part of modern physics
: (of which SRT belongs), except maybe if it can be disproved
: by logic. But I agree that it explains why so many *claim*
: that Einstein was the sole discoverer of SRT, for at that time
: he suggested to have no need for an ether and that is the popular
: philosophy (or should I say mantra or mime?) of today. Harald

I wrote in news:ddfqd9$8vh$1...@news3.bu.edu...
: Could you please explain how would it follow from Poincare or


: Lorentz ideas that, say, radioactive decay works the same in all
: inertial frames (not even talking about things like gravity).

In <42fc9107$1...@epflnews.epfl.ch>, Harry <harald.v...@epfl.ch> wrote:
: I think that Paul Stowe tried to model it somehow, but maybe


: that is not really what you asked. Roughly (from memory):
: - From Poincare we have the PoR: All laws of nature
: (incl. Maxwell's) are independent of our state of inertial motion.
: - From Lorentz+Poincare we have the LT in which t and t' necessarily
: stand for time counters of any natural process in order to obey the PoR.
: Thus from their ideas, radioactive decay must be a process (in the ether)
: that is equally affected by (absolute) speed as any other process.

I do not remember seeing Poincare or Lorentz talking about any phenomena
other than electromagnetic interactions governed by Maxwell equations.
My impression was that the ether was supposed to be the medium just for
these interactions and Lorentz transformations were properties of Maxwell
equations. Of course, Galileo stated a much more general principle but
he used different transformation. Sorry if this is just my ignorance.

Paul Stowe

unread,
Aug 13, 2005, 9:35:54 PM8/13/05
to
On Sat, 13 Aug 2005 03:01:21 +0000 (UTC), cma...@yahoo.com wrote:

>Harry wrote:
>> "Levin" <l...@csb.bu.edu> wrote in message news:ddfqd9$8vh$1...@news3.bu.edu...
>>> In <42f9d355$1...@epflnews.epfl.ch>, Harry <harald.v...@epfl.ch> wrote:
>>> : Metaphysical interpretation isn't part of modern physics
>>> : (of which SRT belongs), except maybe if it can be disproved
>>> : by logic. But I agree that it explains why so many *claim*
>>> : that Einstein was the sole discoverer of SRT, for at that time
>>> : he suggested to have no need for an ether and that is the popular
>>> : philosophy (or should I say mantra or mime?) of today. Harald
>>>
>>> Could you please explain how would it follow from Poincare or
>>> Lorentz ideas that, say, radioactive decay works the same in all
>>> inertial frames (not even talking about things like gravity).
>>
>> I think that Paul Stowe tried to model it somehow, but maybe that
>> is not really what you asked. Roughly (from memory):
>>
>> - From Poincare we have the PoR: All laws of nature (incl. Maxwell's)
>> are independent of our state of inertial motion.
>> - From Lorentz+Poincare we have the LT in which t and t' necessarily
>> stand for time counters of any natural process in order to obey the
>> PoR.
>>
>> Thus from their ideas, radioactive decay must be a process (in the
>> ether) that is equally affected by (absolute) speed as any other
>> process.
>

> I don't know much about the Lorentz-Poincare theory, but perhaps this
> could add another element to your argument.
>

> It's my understanding that although he assumes an ether frame, Lorentz


> showed that inertial frames are not distinguishable from the

> ether frame(s). And consequently, you may conclude that all inertial


> frames are etherframes. And therefore the Maxwell laws hold good in all
> inertial frames, provided they hold in at least one of them.

Maxwell's laws hold good in all inertial frames because he derived from
a physical model that was based upon Faraday's findings. Faraday's
experiments were done on Earth, and as we all know (as did Maxwell),
it is rotating, orbiting the Sun, the Sun's orbiting the galaxy, and
the galaxy is moving. In other words, they are implicitly based solely
upon relative motion between interacting elements. Maxwell clearly
states this. See: http://vacuum-physics.com/Maxwell/maxwell_oplf.pdf
It is a misnomer to claim that these equations need any modification to
account for any other motion. This is a faux pas made by others, who
only considered his equations in isolation, and not the foundational
model from whence they were derived. Maxwell's theory was not just
Maxwell's Equations (as can be seen by the above reference).

> In this theory the "ether frame" turns out to be a very vacuous concept;


> it's only a frame wherein the Maxwell laws apply. Viewing it this way,
> Einstein didn't bring anything new.

True. IIRC Einstein is quoted as saying,

"Remember gentlemen, we haven't disproved the ether, we've
only shown we do not need consider it for these calculations."

> As I said, I don't know too much about this. So... trust but verify.

Indeed. However, if one needs to verify, trust isn't truly there.

Paul Stowe

I.Vecchi

unread,
Aug 14, 2005, 9:10:46 AM8/14/05
to
Levin ha scritto:

..

> I do not remember seeing Poincare or Lorentz talking about any phenomena
> other than electromagnetic interactions governed by Maxwell equations.
> My impression was that the ether was supposed to be the medium just for
> these interactions and Lorentz transformations were properties of Maxwell
> equations. Of course, Galileo stated a much more general principle but
> he used different transformation. Sorry if this is just my ignorance.

In his "La science et l' hypothese" (1902) Poincare' actually devotes
one chapter to the relativity principle. For those who can read French
[1] provides a good survey, with extensive quotes and references. For
example:
"It is irrelevant whether ether really exists. It's a metaphysical
issue ... one day ether will certainly be rejected as useless .. such
hypotheses play only a secondary role. One could eliminate them. It's
only for clarity of explanation that this is not usually done, but this
is the only reason" ([2]).

Regards,

IV

[1] http://www-cosmosaf.iap.fr/Poincare.htm
[2] "Peu nous importe que l'éther existe réellement, c'est l'affaire
des métaphysiciens ... un jour viendra sans doute où l'éther sera
rejeté comme inutile ... Ces hypothèses ne jouent qu'un rôle
secondaire. On pourrait les sacrifier ; on ne le fait pas d'ordinaire
parce que l'exposition y perdrait en clarté, mais cette raison est la
seule." in "La science et l' hypothese" (1902).


Paul Stowe

unread,
Aug 14, 2005, 9:10:45 AM8/14/05
to

One must put these works into historical context. Poincare', Lorentz
and Einstein's works focused primarily on electrodynamics, indeed
Lorentz's work in 1904 and Einsteins in 1905 used the words
electromagnetic/electrodynamics prominantly in their titles. That is
because, in that era the nuclear weak & strong forces were yet unknown
and electromagnetic bonds in the form of Coulombic were known to bind
'ponderable matter' together. They all focused upon the most
fundamental elements known at the time. The intent in all was that
this effect is universal at the most fundamental levels.

Paul Stowe

Douglas Eagleson

unread,
Aug 14, 2005, 1:30:12 PM8/14/05
to
Perspicacious wrote:
> Why isn't the mathematician Henri Poincar=E9

> acknowledged as the true discoverer of the
> special theory of relativity?
>
> http://www-cosmosaf.iap.fr/Poincare-RR3A.htm
> http://arxiv.org/abs/physics/0408077

A mathematical function to transform is not a theory.

That is why.

Levin

unread,
Aug 14, 2005, 1:30:19 PM8/14/05
to
: On Sat, 13 Aug 2005 06:21:52 +0000 (UTC), l...@csb.bu.edu (Levin) wrote:
: > I do not remember seeing Poincare or Lorentz talking about any

: > phenomena other than electromagnetic interactions governed
: > by Maxwell equations. My impression was that the ether was
: > supposed to be the medium just for these interactions and
: > Lorentz transformations were properties of Maxwell equations.
: > Of course, Galileo stated a much more general principle but he
: > used different transformation. Sorry if this is just my ignorance.

In <gmitf1d478sjbji2n...@4ax.com>,
Paul Stowe <p...@acompletelyjunkaddress.net> wrote:
: One must put these works into historical context. Poincare',


: Lorentz and Einstein's works focused primarily on electrodynamics,

: indeed Lorentz's work in 1904 and Einstein's in 1905 used the words
: electromagnetic/electrodynamics prominently in their titles.


: That is because, in that era the nuclear weak & strong forces were yet
: unknown and electromagnetic bonds in the form of Coulombic were known
: to bind 'ponderable matter' together. They all focused upon the most
: fundamental elements known at the time. The intent in all was that
: this effect is universal at the most fundamental levels. Paul Stowe

It was well known that Maxwell equations do not explain all
phenomena (even besides gravity). One did not even need to look
at novel discoveries, such as radioactive decay. It was obvious
that Maxwell equations were insufficient for explaining such
common things as molecular structure of matter. It was Einstein's
rethinking of concepts of space and time that made relativity
applicable to all effects regardless of their physical nature.

P.S.

In <3032.217.124.69.210...@webmail.canonicalscience.com>,
<juanrgo...@canonicalscience.com> wrote:
: There is an increasing interest on the role of Poincare
: on relativity theory, sustaining initial Whittaker statements.
: See A.A. Logunov. Henri Poincare and relativity theory. M: Nauka, 2005.

The interest in Poincare (as well as Lorentz or Hilbert) hardly can be
increased. Their great role in this and many other areas was hardly
unrecognized :-). However, the reference to Logunov explains a bit
the origin of the tone of this thread. From what I know of Logunov
(the President of Moscow University under Soviet regime), the greatest
problem he had with relativity was Einstein's ethnic background.

harald.v...@epfl.ch

unread,
Aug 14, 2005, 4:08:31 PM8/14/05
to

That's correct - although one could (or should?) say that it was
Poincare who showed it for Lorentz' theory, after polishing it up. And
it's exactly like with Newton's theory.

> And consequently, you may conclude that all inertial
> frames are etherframes.

You lost me here, except if you forgot hte word "apparent". Newton
emphasized that the difference between appearance and reality of the
world is crucial for understanding (I agree).

> And therefore the Maxwell laws hold good in all
> inertial frames, provided they hold in at least one of them.

Exactly.

> In this theory the "etherframe" turns out to be a very vacuous concept;
> it's only a frame wherein the Maxwell laws apply. Viewing it this way,
> Einstein didn't bring anything new.

Einstein's was the first to publish a derivation of the Lorentz
transformation from just a few basic laws and principles, without
specifically referring to underlying models. Even Lorentz later adopted
this approach.

Harald

Javier Bezos

unread,
Aug 15, 2005, 12:57:57 PM8/15/05
to
"Perspicacious" <iperspicacious@.com> escribió en el mensaje
news:1123101871.4...@g14g2000cwa.googlegroups.com...
> Why isn't the mathematician Henri Poincaré

> acknowledged as the true discoverer of the
> special theory of relativity?
>
> http://www-cosmosaf.iap.fr/Poincare-RR3A.htm
> http://arxiv.org/abs/physics/0408077

I've recently read Science and Method (1908) by Poincaré
and my impression was that he was clearly on the wrong
road, trying to explain its "new dinamics" in terms
of EM phenomena (eg, with EM self-induction). I was
following this thread and I would like to make a
question.

I think a key point in the PoR as stated by Einstein is:
"light is always propagated in empty space with a definite
velocity c which is independent of the state of motion of
the emitting body". Does Poincaré make a similar statement
in his papers? Please, don't answer that it says something
implying that (which now _a posteriori_ could seem obvious
once Einstein formulated it)---what I would like to know
is if he said _that_ and when.

Thanks
Javier
------------------------------
http://www.texytipografica.com


cma...@yahoo.com

unread,
Aug 15, 2005, 4:23:18 PM8/15/05
to

Please tell me in what respect Einstein's papers are less
Maxwell-based. Everywhere I look, and even in the E=mc^2 paper, I see
references to light and its speed. As far as I know, light is
electromagnetic and is within Maxwell's theory.

Chris

Harry

unread,
Aug 15, 2005, 4:23:31 PM8/15/05
to
"Levin" <l...@csb.bu.edu> wrote in message news:ddnqtt$rpp$1...@news3.bu.edu...

> : On Sat, 13 Aug 2005 06:21:52 +0000 (UTC), l...@csb.bu.edu (Levin) wrote:
> : > I do not remember seeing Poincare or Lorentz talking about any
> : > phenomena other than electromagnetic interactions governed
> : > by Maxwell equations. My impression was that the ether was
> : > supposed to be the medium just for these interactions and
> : > Lorentz transformations were properties of Maxwell equations.
> : > Of course, Galileo stated a much more general principle but he
> : > used different transformation. Sorry if this is just my ignorance.

Certainly not: the ether of Lorentz was the medium for interactions of
matter as well as electromagnetism, for matter was supposed to be
electromagnetic in origin. It was a straightforward extension of Newton's
Absolute Space by incorporating Maxwell's theory and expanding from that:
this assumption of electromagnetic origin was not part of the theory (see
furtherbelow> In <gmitf1d478sjbji2n...@4ax.com>,


> Paul Stowe <p...@acompletelyjunkaddress.net> wrote:
> : One must put these works into historical context. Poincare',
> : Lorentz and Einstein's works focused primarily on electrodynamics,
> : indeed Lorentz's work in 1904 and Einstein's in 1905 used the words
> : electromagnetic/electrodynamics prominently in their titles.
> : That is because, in that era the nuclear weak & strong forces were yet
> : unknown and electromagnetic bonds in the form of Coulombic were known
> : to bind 'ponderable matter' together. They all focused upon the most
> : fundamental elements known at the time. The intent in all was that
> : this effect is universal at the most fundamental levels. Paul Stowe
>
> It was well known that Maxwell equations do not explain all
> phenomena (even besides gravity). One did not even need to look
> at novel discoveries, such as radioactive decay. It was obvious
> that Maxwell equations were insufficient for explaining such
> common things as molecular structure of matter. It was Einstein's
> rethinking of concepts of space and time that made relativity
> applicable to all effects regardless of their physical nature.

In fact it was already Poincare's postulated Principle of Relativity that
achieved that, and Lorentz yielded to it when he drew the conclusion (1904)
that for his theory the following assumption is required:

"I shall suppose that the forces between uncharged
particles, as well as those between such particles and electrons, are
influenced by a translation in quite the same way as the electric forces
in an electrostatic system [...] whatever be the nature of the
particles composing a ponderable body"

> P.S.
>
> In <3032.217.124.69.210...@webmail.canonicalscience.com>,
> <juanrgo...@canonicalscience.com> wrote:
> : There is an increasing interest on the role of Poincare
> : on relativity theory, sustaining initial Whittaker statements.
> : See A.A. Logunov. Henri Poincare and relativity theory. M: Nauka, 2005.
>
> The interest in Poincare (as well as Lorentz or Hilbert) hardly can be
> increased. Their great role in this and many other areas was hardly
> unrecognized :-). However, the reference to Logunov explains a bit
> the origin of the tone of this thread. From what I know of Logunov
> (the President of Moscow University under Soviet regime), the greatest
> problem he had with relativity was Einstein's ethnic background.

That may serve to compensate for the fact that Einstein's ethnic
background worked in favour of his popularity just after the second
world war. It is up to us to reduce unscientific bias.

Harald

Paul Stowe

unread,
Aug 15, 2005, 4:23:11 PM8/15/05
to

This is rather vague and one cannot discern, what in particular,
you are referring to circa 1905. Can you provide specific
example(s)?

> It was Einstein's rethinking of concepts of space and time that
> made relativity applicable to all effects regardless of their
> physical nature.

On this we disagree. The underlying principle is not rethinking
the concepts of space and time but of the very dependent of all
medium based properties upon the the processes that must sustain
them. The abitrarily switching of perspective cannot change or
in any way create any inconsistencies with these. Since c is a
local result of other underlying properties (namely compressibility
and density) it is the natural rate at which information
(perturbations from any equalibrium) can be communicated. Therefore
any medium that is elastic and compressible will be Lorentz covariant
with respect to itself and any processes consisting thereof. This
seems to me to be a very satisfying physical basis upon which to base
the PLR interpretation. In this case Einstein's perspective is a
natural prediction of the premise, and it by definition, must apply
to all physical processes & properties of the medium.

> P.S.
>
>In <3032.217.124.69.210...@webmail.canonicalscience.com>,
> <juanrgo...@canonicalscience.com> wrote:
>: There is an increasing interest on the role of Poincare
>: on relativity theory, sustaining initial Whittaker statements.
>: See A.A. Logunov. Henri Poincare and relativity theory. M: Nauka, 2005.
>
> The interest in Poincare (as well as Lorentz or Hilbert) hardly
> can be increased. Their great role in this and many other areas
> was hardly unrecognized :-). However, the reference to Logunov
> explains a bit the origin of the tone of this thread. From what
> I know of Logunov (the President of Moscow University under Soviet
> regime), the greatest problem he had with relativity was Einstein's
> ethnic background.

I do not know why you bring this up here? But, I totally agree
that Einstein's ethnicity has nothing whatsoever to do with the
issue.

Paul Stowe

cma...@yahoo.com

unread,
Aug 15, 2005, 4:23:57 PM8/15/05
to
harald.v...@epfl.ch wrote:

> cma...@yahoo.com wrote:
> > I don't know much about the Lorentz-Poincare theory, but perhaps this
> > could add another element to your argument.
> >
> > It's my understanding that although he assumes an etherframe, Lorentz
> > showed that inertial frames are not distinguishable from the
> > etherframe(s).
>
> That's correct - although one could (or should?) say that it was
> Poincare who showed it for Lorentz' theory, after polishing it up. And
> it's exactly like with Newton's theory.
>
> > And consequently, you may conclude that all inertial
> > frames are etherframes.
>
> You lost me here, except if you forgot hte word "apparent". Newton
> emphasized that the difference between appearance and reality of the
> world is crucial for understanding (I agree).

I understand what you are saying. If there is such a thing as a unique
true ether frame, then all inertial frames are only apparent ether
frames, this is analogous to the first Scholium in the Principia. But
if there is no such thing as a unique true ether frame, then all
inertial frames are true ether frames. Either way, you reach the very
important conclusion that the speed of light is c in all inertial
frames. Therefore, no matter how we look at it, whether or not there is
a unique true ether frame has no impact on SR. I bet that's why
Poincare said it didn't matter as far as SR was concerned.

>
> > And therefore the Maxwell laws hold good in all
> > inertial frames, provided they hold in at least one of them.
>
> Exactly.
>
> > In this theory the "etherframe" turns out to be a very vacuous concept;
> > it's only a frame wherein the Maxwell laws apply. Viewing it this way,
> > Einstein didn't bring anything new.
>
> Einstein's was the first to publish a derivation of the Lorentz
> transformation from just a few basic laws and principles, without
> specifically referring to underlying models. Even Lorentz later adopted
> this approach.

Yes, but Lorentz said something like: "Einstein postulated what we have
demonstrated painfully...". My words, but the spirit is there. Why, of
course, now that Wiles-Taylor haved painfully shown x^n + y^n != z^n
for n > 2, I can ignore their work and start a new theory from scratch
by postulating x^n + y^n != z^n for n > 2, neither citing them nor
Fermat. This way we won't have to go through the meanders of the
reasoning behind it. And as a bonus, people will think a new genius is
born because it's so non-obvious. They will think I have some direct
connection with God or something. Big deal.

Chris

EF Ferrari

unread,
Aug 15, 2005, 4:51:10 PM8/15/05
to
Levin wrote:

> ...


> In <3032.217.124.69.210...@webmail.canonicalscience.com>,
> <juanrgo...@canonicalscience.com> wrote:
> : There is an increasing interest on the role of Poincare
> : on relativity theory, sustaining initial Whittaker statements.
> : See A.A. Logunov. Henri Poincare and relativity theory. M: Nauka, 2005.
>
> The interest in Poincare (as well as Lorentz or Hilbert) hardly can be
> increased. Their great role in this and many other areas was hardly
> unrecognized :-). However, the reference to Logunov explains a bit
> the origin of the tone of this thread. From what I know of Logunov
> (the President of Moscow University under Soviet regime), the greatest
> problem he had with relativity was Einstein's ethnic background.

I'm not sure about this last point.

In page 40 of the above ref., Logunov says:

<< It is necessary to specially emphasize that the principle of
constancy
of velocity of light, suggested by A. Einstein as the second
independent
postulate, is really a consequence of requirements of the relativity
principle by H. Poincare. (...) To be convinced in this it is
sufficient
to consider requirements of the relativity principle for an elementary
process - propagation of the electromagnetic spherical wave. >>

His argument intending to show that <<this circumstance remained
unnoticed
by A. Einstein>> is exposed in pages 48-49. The crucial (fault) step is
his claim that

<< The requirement of the relativity principle here is reduced to
necessity
that the (spherical) electromagnetic wave in a new inertial reference
system K' has to be also spherical having its center at the origin of
this reference system (coinciding with the origin of the system K at
t=0).>>

The principle of relativity by itself does not imply that a spherical
wave remains spherical in the new coordinates, be it electromagnetic,
sound or bug's wave. Giving to the electromagnetic wave such an
unique attribute is the purpose of Einstein's second postulate.

So it seems probable that "the greatest problem" Logunov "had with
relativity" is not understanding it at all!

Harry

unread,
Aug 16, 2005, 4:36:50 PM8/16/05
to

"Javier Bezos" <see_belo...@yahoo.es> wrote in message
news:1124117339.8...@g49g2000cwa.googlegroups.com...

> "Perspicacious" <iperspicacious@.com> escribió en el mensaje
> news:1123101871.4...@g14g2000cwa.googlegroups.com...
> > Why isn't the mathematician Henri Poincaré
> > acknowledged as the true discoverer of the
> > special theory of relativity?
> >
> > http://www-cosmosaf.iap.fr/Poincare-RR3A.htm
> > http://arxiv.org/abs/physics/0408077
>
> I've recently read Science and Method (1908) by Poincaré
> and my impression was that he was clearly on the wrong
> road, trying to explain its "new dinamics" in terms
> of EM phenomena (eg, with EM self-induction). I was
> following this thread and I would like to make a
> question.

I agree that also IMHO, Poincare also made some strange remarks.

> I think a key point in the PoR as stated by Einstein is:
> "light is always propagated in empty space with a definite
> velocity c which is independent of the state of motion of
> the emitting body". Does Poincaré make a similar statement
> in his papers? Please, don't answer that it says something
> implying that (which now _a posteriori_ could seem obvious
> once Einstein formulated it)---what I would like to know
> is if he said _that_ and when.
>
> Thanks
> Javier

I think he did, but I can't find it at the moment. And does it really
matter? That light speed is independent of the state of motion of the
emitting body and c relative to the ether was already postulated by Maxwell!
In his 1904 presentation, Poincare referred to the Michelson-Morley
experiment and he stated :

"The laws of physical phenomena must be the same for a fixed observer [fixed
relative to the ether or empty space] as for an observer who has a uniform
motion of translation relative to him; so that we have not, and cannot
possibly have, any means of discerning whether we are, or are not, carried
along in such a motion".

Do you really need to hear more? No hindsight is needed to see that the way
Einstein rearranged these statements immediately follows.

Harald

Harry

unread,
Aug 19, 2005, 1:19:25 PM8/19/05
to
- The following addition by me seems to have been lost; thus once more! -

"Javier Bezos" <see_belo...@yahoo.es> wrote in message
news:1124117339.8...@g49g2000cwa.googlegroups.com...

In addition to my other posting about Maxwell's law (accepted by both
Poincare and Einstein) that light emission is independent of the state of
motion of the emitter, I found _that_ literally in Poincare 1898, Poincare
stated about measurements in inertial frames:

" [...] admit that light has a constant speed, and specifically that its
speed is the same in all directions.
Now that's a postulate without which no measurement of that speed could be
attempted."
(translation mine)

BTW, he next elaborated on the definition of simultaneity...

Cheers,
Harald


Harry

unread,
Aug 20, 2005, 6:49:12 AM8/20/05
to
"EF Ferrari" <e...@onda.com.br> wrote in message
news:1124136755.5...@g47g2000cwa.googlegroups.com...SNIP

> His argument intending to show that <<this circumstance remained
> unnoticed
> by A. Einstein>> is exposed in pages 48-49. The crucial (fault) step is
> his claim that
>
> << The requirement of the relativity principle here is reduced to
> necessity
> that the (spherical) electromagnetic wave in a new inertial reference
> system K' has to be also spherical having its center at the origin of
> this reference system (coinciding with the origin of the system K at
> t=0).>>
>
> The principle of relativity by itself does not imply that a spherical
> wave remains spherical in the new coordinates, be it electromagnetic,
> sound or bug's wave. Giving to the electromagnetic wave such an
> unique attribute is the purpose of Einstein's second postulate.
>
> So it seems probable that "the greatest problem" Logunov "had with
> relativity" is not understanding it at all!

- First of all, Logunov had no problem at all with "relativity" - if you
mean the theory of physics that bears that name.

- Second, the PoR as expressed by Poincare was a little more specific
than the way Einstein expressed it. It did imply that that a spherical
wave remains apparently spherical in a moving inertial frame, as follows
(1904):

"The Principle of Relativity, according to which the laws of physical
phenomena must be the same, be it for a fixed observer or for an
observer who is entrained in an uniform translational motion; such that
we don't have and can't have any means to discern if we are, yes or no,
entrained in such a motion." (translation mine)

Thus he included the apparent isotropy of electromagnetic waves in the
PoR.

Harald

Ilja Schmelzer

unread,
Aug 20, 2005, 7:03:47 AM8/20/05
to
"Levin" <l...@csb.bu.edu> schrieb

> Could you please explain how would it follow from Poincare or
> Lorentz ideas that, say, radioactive decay works the same in all
> inertial frames (not even talking about things like gravity).

That's straightforward.

Last not least, as a consequence of his version of SR Poincare has
tried to modify the theory of gravity to make it Lorentz-invariant too.

BTW, about gravity: That the ether is homogeneous and does not
move himself is not nice for the Lorentz ether, ether theorists would
have preferred a moving, inhomogeneous ether. Inhomogeneities
would show up as curved light rays. Instead, SR spacetime was
absolute, it was quite natural for him to be homogeneous.

I guess that the spacetime interpretation would have been as dead
as possible if the influence of gravity on light rays would have been
observed (by accident) before GR. (The appropriate equations would
have been found too, but in the ADM decomposition of the metric
in the form

g^00 sqrt(-g) = rho
g^0i sqrt(-g) = rho v^i
g^ij sqrt(-g) = rho v^i v^j - sigma^ij

with an ether interpretation with density rho, velocity v^i and pressure
tensor sigma^ij, and in harmonic coordinates which, in this
interpretation, give the continuity and Euler equations. (See also
gr-qc/0205035 for such a variant of the theory of gravity)

Ilja

Ilja Schmelzer

unread,
Aug 20, 2005, 7:03:57 AM8/20/05
to
"Levin" <l...@csb.bu.edu> schrieb

> I do not remember seeing Poincare or Lorentz talking about any phenomena
> other than electromagnetic interactions governed by Maxwell equations.

I remember seeing Poincare talking about the necessary modifications
of the theory of gravity, with discussions of a scalar Lorentz-invariant
version of a theory of gravity.

Ilja

Ilja Schmelzer

unread,
Aug 20, 2005, 7:08:21 AM8/20/05
to
"Levin" <l...@csb.bu.edu> schrieb

> It was well known that Maxwell equations do not explain all
> phenomena (even besides gravity). One did not even need to look
> at novel discoveries, such as radioactive decay. It was obvious
> that Maxwell equations were insufficient for explaining such
> common things as molecular structure of matter.

Hm. We know today that it is electromagnetic force which
explains the molecular structure of matter (together with
quantum laws).

At that time it was known only that the explanation of
the structure of matter was unknown.

But nonetheless Lorentz has argued that the molecular
forces may be similar to EM forces, and this may be
used to explain contraction and time dilation.

> However, the reference to Logunov explains a bit
> the origin of the tone of this thread. From what I know of Logunov
> (the President of Moscow University under Soviet regime), the greatest
> problem he had with relativity was Einstein's ethnic background.

Ethnic background was, indeed, an important issue for the
administration under Soviet regime. (And not only of the
Moscow University.)

But, of course, not everybody who follows some part
of Logunov's argumentation about Poincare shares his
views on ethnic questions.

Ilja

Harry

unread,
Aug 20, 2005, 7:08:48 AM8/20/05
to
"Javier Bezos" <see_belo...@yahoo.es> wrote in message
news:1124117339.8...@g49g2000cwa.googlegroups.com...

In addition to my other posting, I found _that_ literally in Poincare 1898:

Javier Bezos

unread,
Aug 20, 2005, 7:10:23 AM8/20/05
to
"Harry" <harald.vanlintel@.ch> escribió en el mensaje
news:4301eea5$1...@epflnews.epfl.ch...

> I agree that also IMHO, Poincare also made some strange remarks.

I've re-read the part on the new dinamics and to
me it's clear Poincaré cannot be credited as _the_
author of the SR. On the other hand, the article in the
Britannica about Relativity says _nothing_ about Poincaré,
which is tremendously unfair. Perhaps a middle point is the
right answer (as in Pauli's Theory or Relativity, which
credites the work of Lorenz and Poincaré as fundamental).

> > in his papers? Please, don't answer that it says something
> > implying that (which now _a posteriori_ could seem obvious

> > once Einstein formulated it) [...]

> Do you really need to hear more?

Yes, I do!

> No hindsight is needed to see that the way
> Einstein rearranged these statements immediately follows.

But not that... :-)

Let's think...

Maxwell did not discover the laws of EM. They are based in laws
which were well know before he wrote his equations. Further,
he introduced a displacement current in the ether which is
known to be false.

Newton did not discover the laws of Mechanics...

Then, why are they so famous? Because they were able to
make obvious what for many people was not, and to reduce
to basic principles what by then was a large amount
of unconnected laws. This was exactly what Einstein
did---now, we can understand the Lorentz and Poincaré ideas
in a consistent framework. If currenly could seem Lorentz
and Poincaré discovered the SR as we know it, is just
because Einstein (and Minkowski shortly after, taking ideas
from Poincaré) was able to synthetize and to put order in
all these laws so that we **a posteriori** could find them
obvious and consistent (thus, your "immediately follows" is
essentially a tribute to Einstein). This achievement cannot
be credited to neither Lorentz nor Poincaré.

Interestingly, when trying to demonstrate Lorenz and Poincaré
were the real authors of SR, their ideas are compared to
those of Einstein, thus recognizing implicitly that his 1905
paper is a milestone. Further, and even more interesting, is
the fact their ideas about how to apply the LT to Mechanics
(as in Poincaré's Science and Method) are usually "concealed"
because they do not fit well in Einstein's SR and are plenty
of "curious" interpretations (and realizing that LT should
not be interpreted because they are basic principles is
another achievement of Einstein).

Javier
-----------------------------
http://www.texytipografia.com

Javier Bezos

unread,
Aug 20, 2005, 12:06:29 PM8/20/05
to
Harry:

> In addition to my other posting about Maxwell's law (accepted by both
> Poincare and Einstein) that light emission is independent of the state of
> motion of the emitter, I found _that_ literally in Poincare 1898, Poincare
> stated about measurements in inertial frames:
>
> " [...] admit that light has a constant speed, and specifically that its
> speed is the same in all directions.
> Now that's a postulate without which no measurement of that speed could be
> attempted."
> (translation mine)

Yes, I knew that. However, he does not say in relation
to what. It was commonly accepted it was in relation to
the ether. The problem was if the ether was in rest or
moved with bodies. As of 1908 Poincaré was still concerned
with this issue and even accepts the _ad hoc_ partial
drag as a demonstrated fact (by then, the drag coefficient
was explained in terms of the SR by Laue, including an
additional term which was proven to be true by Zeeman
shortly after).

So may question remains.

[BTW, it seems a message of mine has been lost.]

Javier
-----------------------------
http://www.texytipografia.com


Levin

unread,
Aug 21, 2005, 12:03:09 PM8/21/05
to
This Einstein bashing still goes on! Was not it investigated in the 30s
to exhaustion by German scientists and later by their other followers?
It is not a renewed interest in Poincare, Lorentz, or Hilbert (which
never diminished so that it could be renewed). Is it a renewed lack of
interest in Einstein? But for some (like Logunov) this lack of interest
in some of their colleagues is a life-lasting passion, so nothing to be
renewed either ... If you want to keep beating at this please discuss
things that are actually credited to Einstein, not the invention of the
main math tools of relativity or of either of its two basic postulates ...

Let me try to divert this to more technical issues.

Could somebody please explain why was Michelson-Morley test needed?
Were not contemporary clocks good enough so that Galilean dependence
of the speed of light on the motion of the emitting/reflecting or
receiving bodies could be excluded by astronomical data?

Javier Bezos

unread,
Aug 22, 2005, 5:11:05 AM8/22/05
to
<cmaj10@.com> wrote:

> Yes, but Lorentz said something like: "Einstein postulated what we have
> demonstrated painfully...". My words, but the spirit is there. Why, of

I only have a translation into Spanish and not the original, and
that's more or less what he said. However, it's not _all_ what
he said--he added his deduction was not fully satisfactory and
that Einstein has to be credited for showing that experiments
like those of Michelson, Rayleigh and Brace are not explained
as a fortuitous compensation of opposite effects but they are
the manifestation of a fundamental and general principle. It's
in his book The Theory of Electrons (1909, sec. 194, based on
a series of lectures given in 1906). Thus, Lorentz himself
says Einstein was the discoverer of the SR and emphasizes its
importance and novelty.

BTW, in other message <markwh04@.com> wrote:

> The reason Poincare' is not granted as the true discoverer of special
> relativity is that he, himself, disavowed such claim, citing Einstein
> as the discoverer of the theory.

I presume this is a mistake, because AFAIK Poincar=E9 never
cited Einstein in relation to the PoR (why?). Remember
Poincar=E9 was dead on 1912, only seven years after Einstein's
paper. I think it's a wrong attribution to Poincar=E9 instead
of Lorentz.

Javier
-----------------------------
http://www.texytipografia.com

Paul Stowe

unread,
Aug 22, 2005, 5:15:12 AM8/22/05
to
On Sun, 21 Aug 2005 16:03:09 +0000 (UTC), l...@csb.bu.edu (Levin) wrote:

> This Einstein bashing still goes on!

Bashing? I doubt the moderators would allow such ad-hominems.
No, the questions raised herein were based upon historical
documentation of factual evidence, found within the disputed
authors papers, not ad-hominem attacks. I seems that only you
keep raising this issue here. Separate the message from the
messengers. You know the old saw, "don't shoot the messenger'\".

> Was not it investigated in the 30s to exhaustion by German scientists
> and later by their other followers?

Can you point to refernces?

> It is not a renewed interest in Poincare, Lorentz, or Hilbert (which
> never diminished so that it could be renewed). Is it a renewed lack
> of interest in Einstein?

Huh? I doubt there exists any lack of interest in Einstein.

> But for some (like Logunov) this lack of interest in some of their
> colleagues is a life-lasting passion, so nothing to be renewed either ...

You keep bringing up Logunov, but others historically have raised
the same issue of priorty, another example, Sir Edmund Whittaker.

> If you want to keep beating at this please discuss things that are
> actually credited to Einstein, not the invention of the main math
> tools of relativity or of either of its two basic postulates ...

If 'you' think or agree Einstein did not 'invent' either the
mathematics of SR, nor either of its two basic postulates, then
the question is, why should he get credit for it? Should not the
credit for the founding of the theory go to the people that
were the one(s) that actually first did? Just curious? Because
this seems to be the basic thrust of the thread(s) you're
complaining about.

> Let me try to divert this to more technical issues.

OK,

> Could somebody please explain why was Michelson-Morley test needed?

Needed? The MMX isn't 'needed' for SR, only the assumption that
Maxwell's equations as modeled in his 1861/62 paper are basically
correct as defined, and c is a natural property at which
perturbations are transmitted. The result of the MMX then become
a straight forwardprediction of this. However, experimental
history unfolded differently. The result OF the MMX brought the
issue to the fore and the sequence was reversed.

> Were not contemporary clocks good enough so that Galilean
> dependence of the speed of light on the motion of the
> emitting/reflecting or receiving bodies could be excluded by
> astronomical data?

What astronomical data, circa 1880-1904?

Paul Stowe

cma...@yahoo.com

unread,
Aug 22, 2005, 5:26:57 AM8/22/05
to
Levin wrote:
> This Einstein bashing still goes on! Was not it investigated in the 30s

> to exhaustion by German scientists and later by their other followers?
> It is not a renewed interest in Poincare, Lorentz, or Hilbert (which
> never diminished so that it could be renewed). Is it a renewed lack of
> interest in Einstein? But for some (like Logunov) this lack of interest

> in some of their colleagues is a life-lasting passion, so nothing to be
> renewed either ... If you want to keep beating at this please discuss

> things that are actually credited to Einstein, not the invention of the
> main math tools of relativity or of either of its two basic postulates ...

Stop the politics, and bring solid arguments that support his priority
over Lorentz, Poincare and Hilbert.

> Let me try to divert this to more technical issues.

The usual tactics when one has no arguments.

Chris

> Could somebody please explain why was Michelson-Morley test needed?

Nick Maclaren

unread,
Aug 24, 2005, 10:40:29 AM8/24/05
to

In article <unihg110p8peqg8tv...@4ax.com>,

Paul Stowe <p...@acompletelyjunkaddress.net> writes:
|> On Sun, 21 Aug 2005 16:03:09 +0000 (UTC), l...@csb.bu.edu (Levin) wrote:
|>
|> > Could somebody please explain why was Michelson-Morley test needed?
|>
|> [ Reply snipped ]

|>
|> > Were not contemporary clocks good enough so that Galilean
|> > dependence of the speed of light on the motion of the
|> > emitting/reflecting or receiving bodies could be excluded by
|> > astronomical data?
|>
|> What astronomical data, circa 1880-1904?

Timing of the eclipses of Jupiter's moons, for a start. Much
older.

The answer to his question is that a direct experiment is many
times more trustworthy than an indirect one. With either, if
ANY of the assumptions upon which you are basing your deductions
are incorrect, your conclusion is unreliable. And there are a
lot more and more sensitive assumptions in an indirect experiment.

For example, what about eddy currents in the aether around
Jupiter?

This is the reason that the cosmological 'proofs' of general
relativity at high space-time stresses are so much porcine
cleansing fluid. There are so many alternative ways of
explaining the observations that they are at best not-disproofs.
No, I am not bashing Einstein - I am bashing his more fanatical
followers.


Regards,
Nick Maclaren.

Harry

unread,
Aug 25, 2005, 4:43:08 AM8/25/05
to
"Javier Bezos" <see_belo...@yahoo.es> wrote in message
news:1124536551.8...@g49g2000cwa.googlegroups.com...

> Harry:
>
> > In addition to my other posting about Maxwell's law (accepted by both
> > Poincare and Einstein) that light emission is independent of the state
of
> > motion of the emitter, I found _that_ literally in Poincare 1898,
Poincare
> > stated about measurements in inertial frames:
> >
> > " [...] admit that light has a constant speed, and specifically that its
> > speed is the same in all directions.
> > Now that's a postulate without which no measurement of that speed could
be
> > attempted."
> > (translation mine)
>
> Yes, I knew that. However, he does not say in relation to what.

Yes he did and as I wrote above: he discussed observations in inertial
frames that move relative to each other.

> It was commonly accepted it was in relation to the ether.

You missed the point:
- While he emphasized that wrt the ether *no speed can be measured*, and
that the ether concept perhaps would be abolished;
- Here he discussed *measurements in inertial frames*: in order to even be
able to define a system of reference, light speed must be assumed to be the
same in all directions *in the used inertial frame* (just read that text
again...)

> The problem was if the ether was in rest or moved with bodies.

I have not seen that dragged ether concept in the writings of Poincare;
instead he advocated the stationary ether concept of Lorentz, as he also
demonstrated in the thought experiments that I know of.
Thus: reference please!

> As of 1908 Poincaré was still concerned
> with this issue and even accepts the _ad hoc_ partial
> drag as a demonstrated fact (by then, the drag coefficient
> was explained in terms of the SR by Laue, including an
> additional term which was proven to be true by Zeeman
> shortly after).

Reference?

Harald

> So may question remains.

Javier Bezos

unread,
Aug 26, 2005, 2:57:24 AM8/26/05
to
Harry wrote:

> > As of 1908 Poincar=E9 was still concerned


> > with this issue and even accepts the _ad hoc_ partial
> > drag as a demonstrated fact (by then, the drag coefficient
> > was explained in terms of the SR by Laue, including an
> > additional term which was proven to be true by Zeeman
> > shortly after).
>
> Reference?

Science and Method. At the end of the section about
aberration (chap. 2) he says on Fizeau's experiments
and the ether: "Those experiments have confirmed the
the Fresnel's hypothesis of partial drag". To me it's
a mistery why Poincar=E9 returned to the ether after
almost rejecting it. As to the Laue's paper, Ann.
Phys., 1907, 989, but that result was, IIR, already
found by Lorentz using quite different reasonings.

Javier
-----------------------------
http://www.texytipografia.com

Harry

unread,
Aug 26, 2005, 5:23:58 AM8/26/05
to

"Javier Bezos" <see_belo...@yahoo.es> wrote in message
news:1124986274.3...@g47g2000cwa.googlegroups.com...

Thanks! I doubt that that corresponded to Lorentz' stationary ether concept.
Perhaps Poincare found that model at that moment too appealing to reject:
sometimes different, incompatible models are used for convenience,
eventhough we know that it can't be the final answer.
Nowadays similarly, many consider light to be a kind of bullet as well as a
kind of wave, depending on the situation (on purpose I don't use "particle",
for a particle is not a model).

Harald


EF Ferrari

unread,
Aug 27, 2005, 3:25:49 AM8/27/05
to

Unfortunately that's wrong.

Take a spherical wave propagating with a speed v,

v^2 t^2 - x^2 - y^2 - z^2 = 0

is true in a rest frame K,

but

v^2 t'^2 - x'^2 - y'^2 - z'^2 = 0

is not true in a moving frame K' except if v = c.

Thus, the principle of relativity does not imply as much
as you and Logunov say and need to dismiss Einstein's second
postulate, which is necessary to stablish the validity of
the exception v = c.

harald.v...@epfl.ch

unread,
Aug 29, 2005, 10:49:42 AM8/29/05
to

Indeed - that implies that v-wave *must* be c in order for the PoR to
be valid: otherwise we *would* have a means "to discern if we are, yes


or no, entrained in such a motion".

BTW I referred above only to the apparent isotropy, but thanks for this
useful additional remark.

Harald

cma...@yahoo.com

unread,
Aug 30, 2005, 4:09:24 PM8/30/05
to
> Thus, the principle of relativity does not imply as much
> as you and Logunov say and need to dismiss Einstein's second
> postulate, which is necessary to stablish the validity of
> the exception v = c.

Yes, v^2 t'^2 - x'^2 - y'^2 - z'^2 = 0
is true in a moving frame K' even if v < c.

In almost every physical law you must add an implicit "all other things
being equal" condition.

"All other things being equal" in case of a wave means: the source as
well as its medium are at rest in K', just as they are in K.

Now you'll say: what about light then? Isn't the ether not at rest in
K'? Well that's right, but the medium of "apparent light" is not ether:
it is the moving space. And the moving space is at rest in K'. So all
conditions are satisfied. The ether's only role in this theory is to
act on the moving "space-time" by "contracting-dilating" it.

Whether or not the ether truly exists doesn't really matter in
Poincare-Lorentz. They are simply being less bold than Einstein, they
allow it the possibility of existing. As I see it, the true difference
between these two theories is this: Poincare-Lorentz say that
relatively moving frames experience "space-time" contraction-dilation,
and therefore the speed of light (true or apparent) is the same in
every inertial frame. Whereas Einstein proceeds in the opposite
direction: the speed of light (true or apparent, but always true in his
opinion) is the same in every inertial frame, and therefore relatively
moving frames experience "space-time" contraction-dilation.

Who's proceeding in the right order? It's really a question of the
chicken and the egg. In my opinion, both ways are probably right. It
may just be that "space-time" contraction-dilation and constancy of
light speed are two facets of the same thing.

At first sight it might be more satisfying to think of constancy of
light speed as being the cause of relativistic phenomena. But then, is
constancy of light speed really more intuitive than space-time
contraction-dilation?

One thing is ABSOLUTELY certain, both theories predict exactly the same
phenomena. If you can disprove this, I'll venerate you.

Chris

Cl.Massé

unread,
Oct 12, 2005, 4:18:08 AM10/12/05
to
"Harry" <harald.v...@epfl.ch> a écrit dans le message de news:
430aedf1$1...@epflnews.epfl.ch...

> You missed the point:
> - While he emphasized that wrt the ether *no speed can be measured*, and
> that the ether concept perhaps would be abolished;
> - Here he discussed *measurements in inertial frames*: in order to even be
> able to define a system of reference, light speed must be assumed to be
> the same in all directions *in the used inertial frame* (just read that
> text again...)

As Poincaré says that the light speed "can't be measured", it is obviously
not the same concept as in Einsteins relativity. In the latter, the
definition of the measurement procedure of space and time imply that the so
measured light speed is c in every frame of reference. Poincaré therefore
lacks the gist of Einsteins relativity, which is what we do exactly when we
measure space and time. He had only flashes and lucky hits, but never saw
the whole picture. Although he had almost all the material in his whole
writings, they were always associated with incorrect material in any
particular paper. That's why the purported Poincaré's theory of relativity
doesn't exist, and consequently why nobody here is able to give a sketch of
it.

--
~~~~ clmasse on free F-country
Liberty, Equality, Profitability.

Harry

unread,
Oct 14, 2005, 3:07:42 AM10/14/05
to
"Cl.Massé" <to...@tata.ti> wrote in message
news:434aa1c8$0$9731$626a...@news.free.fr...

> "Harry" <harald.v...@epfl.ch> a écrit dans le message de news:
> 430aedf1$1...@epflnews.epfl.ch...
>
> > You missed the point:
> > - While he emphasized that wrt the ether *no speed can be measured*, and
> > that the ether concept perhaps would be abolished;
> > - Here he discussed *measurements in inertial frames*: in order to even
be
> > able to define a system of reference, light speed must be assumed to be
> > the same in all directions *in the used inertial frame* (just read that
> > text again...)
>
> As Poincaré says that the light speed "can't be measured", it is obviously
> not the same concept as in Einsteins relativity.

Of course, *one-way* light speed differes from two-way light speed, as
it can't be verified in relativity and thus no true measurement of it is
possible. By 1905 Poincare had already explained all that, consequenctly
Einstein defined in his paper "light speed" as the average two-way
speed. This was known in 1905 and Einstein's paper is online, so why
don't people know it in 2005?

> In the latter, the
> definition of the measurement procedure of space and time imply that the
so
> measured light speed is c in every frame of reference.

That was first of all explained by Poincare who was the very one who
started that measurement procedure and already discussed the
implications. Einstein merely showed that he understood that. But it's
useless to repeat all explanations again and again.

> Poincaré therefore
> lacks the gist of Einsteins relativity, which is what we do exactly when
we
> measure space and time. He had only flashes and lucky hits, but never saw
> the whole picture.

To the contrary: it was Poincare who presented the whole picture in
overview papers (some of which Einstein studied) as well as at
conferences.

> Although he had almost all the material in his whole
> writings, they were always associated with incorrect material in any
> particular paper. That's why the purported Poincaré's theory of
relativity
> doesn't exist, and consequently why nobody here is able to give a sketch
of
> it.

That explanation can't be right: few papers were free of errors, and
it's not better nowadays. Also Einstein's 1905 paper was not without
error. Anyway, plausible reasons for lack of acknowledgement have
already been given, there is no need for other.

And surely sketches have been given, I wonder what you want more? Here's
my sketch in a nutshell: Poincare's theory of the New Mechanics is that
of Lorentz, but with his PoR as keystone (just a different emphasis).
Thus, the New Mechanics according to Poincare is the PoR with Lorentz'
theory and the LT as solution. But since he published his sketch in a
note in 1905, I wonder why you ask for it...

Best regards,
Harald

Juan R.

unread,
Oct 14, 2005, 3:13:39 AM10/14/05
to
Cl.Massé wrote:

> That's why the purported Poincaré's theory of relativity
> doesn't exist, and consequently why nobody here is able to give a sketch of
> it.

Unfortunately you continue doing 'extreme' claims with none solid
basis. Evidence of why Poincaré obtained special relativity before
Einstein was extensively discussed here. Only myself already presented
more than 15 references on the topic, many of them very recent with new
data on the topic of why Poincaré anticipated Einstein.

I will not repeat again why you are wrong, simply I will cite to Pauli,
brilliant physicist -Nobel laureate- and well-versed in relativity.
Initially, Pauli studied only Einstein theory and in his famous 1921
book on relativity Pauli claimed that Einstein was the father of
relativity over others authors like Lorentz. In 1955, Pauli changed his
mind after studying Poincaré theory:

"Both Einstein and Poincare, took their stand on the preparatory work
of H.A. Lorentz, who had already come quite close to the result,
without however quite reaching it. In the agreement between the results
of the methods followed independently of each other by Einstein and
Poincare I discern a deeper significance of a harmony between the
mathematical method and analysis by means of conceptual experiments
(Gedankenexperimente), which rests on general features of physical
experience."

Pauli says *agreement*, whereas you continue to claim that "there is
not Poincaré relativity", which is -sorry to say this- completely
absurd.

Others brilliant physicists have expressed similar views -contrary to
your own personal view-. The list is excesively large as for being
cited here.

"The special theory of relativity resulted from the joint efforts of a
group of great researchers: Lorentz, Poincare, Einstein, Minkowski"
(Max Born).

"On the contrary, Poincare achieved total invariance of the equations
of electrodynamics and formulated the relativity postulate - a term
first introduced by him" (H. Lorentz)

"I remind on these ideas by Poincare because they are closed to methods
later used by Minkowski and other scientists to easing mathematical
actions in the theory of relativity." (H. Lorentz)

Etc.

Juan R.

Center for CANONICAL |SCIENCE)

Javier Bezos

unread,
Oct 15, 2005, 8:02:36 AM10/15/05
to
Juan R. wrote:

> I will not repeat again why you are wrong, simply I will cite to Pauli,
> brilliant physicist -Nobel laureate- and well-versed in relativity.
> Initially, Pauli studied only Einstein theory and in his famous 1921
> book on relativity Pauli claimed that Einstein was the father of
> relativity over others authors like Lorentz. In 1955, Pauli changed his

> mind after studying Poincar=E9 theory:


>
> "Both Einstein and Poincare, took their stand on the preparatory work
> of H.A. Lorentz, who had already come quite close to the result,
> without however quite reaching it. In the agreement between the results
> of the methods followed independently of each other by Einstein and
> Poincare I discern a deeper significance of a harmony between the
> mathematical method and analysis by means of conceptual experiments
> (Gedankenexperimente), which rests on general features of physical
> experience."

It would be interesting to know the reference to see the
context (1955 was an interesting year for several reasons).
As it has been shown here repeated times you very often
truncate the citations in order to change the true
intentions of theirs authors.

Read the Preface of the Theory of Relativity where Pauli
categorically claims: "Eintein, the creator of the theory
of relativity". And please, read the _whole_ Preface since
it repeats this claim several times (the last paragraph,
for example).

Javier
-----------------------------
http://www.texytipografia.com

Cl.Massé

unread,
Oct 16, 2005, 6:07:19 PM10/16/05
to
"Juan R." <juanrgo...@canonicalscience.com> a écrit dans le message de
news: 1129202756....@z14g2000cwz.googlegroups.com...

> Unfortunately you continue doing 'extreme' claims with none solid
> basis. Evidence of why Poincaré obtained special relativity before
> Einstein was extensively discussed here. Only myself already presented
> more than 15 references on the topic, many of them very recent with new
> data on the topic of why Poincaré anticipated Einstein.

Well, you continue to present irrelevant material. The paternity of
relativity doesn't depends on the opinion of other physicists, however
great they may be. It depends on the writing of the presumed father.
Like physics, history rests on data, not on theories, but there are no
new data, whatever the new theories may be.

Cl.Massé

unread,
Oct 16, 2005, 6:07:38 PM10/16/05
to
I wrote:

> > As Poincaré says that the light speed "can't be measured", it is
> > obviously not the same concept as in Einsteins relativity.

"Harry" <harald.v...@epfl.ch> a écrit dans le message de news:
434ce262$1...@epflnews.epfl.ch...

> Of course, *one-way* light speed differes from two-way light speed, as
> it can't be verified in relativity and thus no true measurement of it is
> possible.

Alas yes. The synchronization of clocks enables to independently
measure the one-way speed, since it uses a two-way beam, and the result
is then c in both directions. This procedure defines the physical space
and time, in the framework of Einsteins relativity theory. Therefore,
Poincaré hadn't this physical space, and was still musing while Einstein
hit the solution.

(btw, in the classical ether theory, the two-way light speed isn't c.
That's why the displacement of the interference fringes in the MM experiment
was predicted.)

> By 1905 Poincare had already explained all that, consequenctly
> Einstein defined in his paper "light speed" as the average two-way
> speed.

Therefore no. Einstein postulated the one-way light speed, and could
validate his theory by measuring it and getting c (which to my knowledge
nobody did.)

> To the contrary: it was Poincare who presented the whole picture in
> overview papers (some of which Einstein studied) as well as at
> conferences.

No single paper of Poincaré contains the whole picture, you have to
separate the wheat from the chaff in several papers and put it together.
Unless you have the explicit reference of this mysterious minutes or
paper...

> That explanation can't be right: few papers were free of errors, and
> it's not better nowadays.

It isn't a matter of error, it is a matter of getting all the pieces of
the jigsaw puzzle in place. Poincaré had most of the pieces, Einstein
had the picture.

> And surely sketches have been given, I wonder what you want more? Here's
> my sketch in a nutshell: Poincare's theory of the New Mechanics is that
> of Lorentz, but with his PoR as keystone (just a different emphasis).
> Thus, the New Mechanics according to Poincare is the PoR with Lorentz'
> theory and the LT as solution. But since he published his sketch in a
> note in 1905, I wonder why you ask for it...

How are the Lorentz theory and the PoR articulated together? With which
interpretation? With which predictions? It's all what I want to know.

Harry

unread,
Oct 17, 2005, 2:56:36 PM10/17/05
to
"Cl.Massé" <to...@tata.ti> wrote in message
news:43513fa2$0$4817$626a...@news.free.fr...

I'm quite sure that Juan meant with "new", new in the sense of data-mining,
just as a recent archaeological find is "new" but at the same time very old
and thus in that sense not new at all. See for example:
http://www.mpiwg-berlin.mpg.de/litserv/diss/janssen_diss/Chapter3.pdf

IOW, that last objection is itself completely irrelevant.
But for reasons of time, I'll stop trying to avoid that this discussion will
end with another misconception.

Best regards,
Harald

Harry

unread,
Oct 17, 2005, 2:57:09 PM10/17/05
to
"Cl.Massé" <to...@tata.ti> wrote in message
news:43513fa5$0$4817$626a...@news.free.fr...

> I wrote:
>
> > > As Poincaré says that the light speed "can't be measured", it is
> > > obviously not the same concept as in Einsteins relativity.
>
> "Harry" <harald.v...@epfl.ch> a écrit dans le message de news:
> 434ce262$1...@epflnews.epfl.ch...
>
> > Of course, *one-way* light speed differes from two-way light speed, as
> > it can't be verified in relativity and thus no true measurement of it is
> > possible.
>
> Alas yes. The synchronization of clocks enables to independently
> measure the one-way speed, since it uses a two-way beam, and the result
> is then c in both directions. This procedure defines the physical space
> and time, in the framework of Einsteins relativity theory. Therefore,
> Poincaré hadn't this physical space, and was still musing while Einstein
> hit the solution.

By thus inversing the logic you disagree with Einstein and Poincare:
according to both, clocks are sychronized in order to *"make"* light
speed the same in all diretions *relative to the used frame* (relative
to other inertial frames it becomes c-v, as Einstein already explained
in 1905). Anyway: as they agreed on this issue, it can't be an argument
that is relevant for this thread.

> (btw, in the classical ether theory, the two-way light speed isn't c.
> That's why the displacement of the interference fringes in the MM
experiment
> was predicted.)
>
> > By 1905 Poincare had already explained all that, consequenctly
> > Einstein defined in his paper "light speed" as the average two-way
> > speed.
>
> Therefore no. Einstein postulated the one-way light speed, and could
> validate his theory by measuring it and getting c (which to my knowledge
> nobody did.)

Please *study* his paper (and not just the intro!):
http://www.fourmilab.com/etexts/einstein/specrel/www/

> > To the contrary: it was Poincare who presented the whole picture in
> > overview papers (some of which Einstein studied) as well as at
> > conferences.
>
> No single paper of Poincaré contains the whole picture,

Very right - at least you agree about that.
Evidently Einstein's 1905 paper was the first good overview.

> you have to
> separate the wheat from the chaff in several papers and put it together.
> Unless you have the explicit reference of this mysterious minutes or
> paper...

What mysterious minutes? I must have missed something interesting!

> > That explanation can't be right: few papers were free of errors, and
> > it's not better nowadays.
>
> It isn't a matter of error, it is a matter of getting all the pieces of
> the jigsaw puzzle in place. Poincaré had most of the pieces, Einstein
> had the picture.

The picture that Poincare had, has already been shown by others in this
thread to be sufficiently described in his papers. From that, along with
the (at that time) complete disregard of Einstein for Poincare's work,
it's understandable that Poincare chose to omit referal to Einstein on
the subject of special relativity (what was your explanation for that? I
don't recall hearing it.)

> > And surely sketches have been given, I wonder what you want more? Here's
> > my sketch in a nutshell: Poincare's theory of the New Mechanics is that
> > of Lorentz, but with his PoR as keystone (just a different emphasis).
> > Thus, the New Mechanics according to Poincare is the PoR with Lorentz'
> > theory and the LT as solution. But since he published his sketch in a
> > note in 1905, I wonder why you ask for it...
>
> How are the Lorentz theory and the PoR articulated together? With which
> interpretation? With which predictions? It's all what I want to know.

They are neatly articulated together in the well-known overview that
Einstein gave immediately following their papers, in 1905, but with the
"hic" that Lorentz at that time still didn't understand the physical
meaning of his "local time" as well as the symmetry of the LT, while
Poincare erroneously assumed that Lorentz understood these things.

Predictions:
- Lorentz correctly predicted the "mass increase" of accelerated
electrons and protons
- Poincare showed how with the new theory any inertial reference frame
is equivalent for observations, which effectively is a prediction that
all inertial frames are equivalent for experiments just as it was in
Newtonian mechanics.

- Einstein added two clock experiment predictions, one correct and one
flawed.

(Did I overlook a prediction?)

-> New thought, for readers to "shoot down" if they can:

According to me, Einstein's clock rate "application" of a round going
clock does not directly follow from his (or Mileva's) LT derivation:
it's far from directly evident from that derivation concerning two
inertial frames, that time dilation also should apply to a clock that is
not resting in an inertial frame. However, such *does* immediately
follow from the physical interpretation of Lorentz's theory by Poincare,
according to which in any inertial frame, "local time" is affected by
speed as if the frame is at rest in the ether!

Harald

harry

unread,
Oct 18, 2005, 8:32:45 PM10/18/05
to

Likely the following is relevant:

http://www.questia.com/PM.qst?a=o&d=88787417

Cheers,
Harald

PS Google is great

Cl.Massé

unread,
Oct 19, 2005, 8:36:31 PM10/19/05
to
I wrote:

> > Alas yes. The synchronization of clocks enables to independently
> > measure the one-way speed, since it uses a two-way beam, and the result
> > is then c in both directions. This procedure defines the physical space
> > and time, in the framework of Einsteins relativity theory. Therefore,
> > Poincaré hadn't this physical space, and was still musing while Einstein
> > hit the solution.

"Harry" <harald.v...@epfl.ch> a écrit dans le message de news:
4353a587$1...@epflnews.epfl.ch...

> By thus inversing the logic you disagree with Einstein and Poincare:
> according to both, clocks are sychronized in order to *"make"* light
> speed the same in all diretions *relative to the used frame* (relative
> to other inertial frames it becomes c-v, as Einstein already explained
> in 1905). Anyway: as they agreed on this issue, it can't be an argument
> that is relevant for this thread.

They don't agree since (in the same paper) Poincaré says the postulated
light speed c can't be measured. The theories are necessarily different,
since they have different experimental sides.

> Please *study* his paper (and not just the intro!):
> http://www.fourmilab.com/etexts/einstein/specrel/www/

Never assume.

> > No single paper of Poincaré contains the whole picture,

> Very right - at least you agree about that.
> Evidently Einstein's 1905 paper was the first good overview.

Call it overview if you like, call a bunch of stones and timbers a house if
you like, but Poincaré never saw the whole picture, and always associated a
right piece with a wrong one. Poincaré had only a heap of red, green, and
blue spots, Einstein had a photograph.

> The picture that Poincare had, has already been shown by others in this
> thread to be sufficiently described in his papers.

Sorry, I couldn't read the sketch on this group. Write it or else it is
futile to continue this discussion.

Harry

unread,
Oct 20, 2005, 11:41:54 AM10/20/05
to

"Cl.Massé" <to...@tata.ti> wrote in message
news:43568389$0$16538$626a...@news.free.fr...

> I wrote:
>
> > > Alas yes. The synchronization of clocks enables to independently
> > > measure the one-way speed, since it uses a two-way beam, and the
> > > result is then c in both directions. This procedure defines the
> > > physical space and time, in the framework of Einsteins relativity
> > > theory. Therefore, Poincaré hadn't this physical space, and was
> > > still musing while Einstein hit the solution.
>
> "Harry" <harald.v...@epfl.ch> a écrit dans le message de news:
> 4353a587$1...@epflnews.epfl.ch...
>
> > By thus inversing the logic you disagree with Einstein and Poincare:
> > according to both, clocks are sychronized in order to *"make"* light
> > speed the same in all diretions *relative to the used frame* (relative
> > to other inertial frames it becomes c-v, as Einstein already explained
> > in 1905). Anyway: as they agreed on this issue, it can't be an argument
> > that is relevant for this thread.
>
> They don't agree since (in the same paper) Poincaré says the postulated
> light speed c can't be measured. The theories are necessarily different,
> since they have different experimental sides.

That's wrong, as I explained to you here above. And I explained to you here
above that according to SRT (Poincare or Einstein, doesn't matter), OWLS
(one-way light speed) can't be truly measured. Therefore I won't bother
anymore. However, if you or a bystander wants to know more, you may consult
the sci.physics.relativity group where the same has just now been discussed
today (again, the topic comes back regularly due to newcomers):

http://groups.google.com/group/sci.physics.relativity/browse_frm/thread/311f43a2d7dcb3d2/d207e073ca7b7b90?lnk=st&q=group:sci.physics.relativity+author:tom+author:roberts&rnum=2&hl=en#d207e073ca7b7b90
("it's not the people who understand SR who are confused. Since OWLS
measurements can yield any value, the people who keep insisting that they be
made have deluded themselves.")

> > Please *study* his paper (and not just the intro!):
> > http://www.fourmilab.com/etexts/einstein/specrel/www/
>
> Never assume.

? Apparently you *still* didn't see SRT's definition of *light speed*! ...
It's only a mouse button click away...

> > > No single paper of Poincaré contains the whole picture,
>
> > Very right - at least you agree about that.
> > Evidently Einstein's 1905 paper was the first good overview.
>
> Call it overview if you like, call a bunch of stones and timbers a
> house if you like, but Poincaré never saw the whole picture, and
> always associated a right piece with a wrong one. Poincaré had only a
> heap of red, green, and blue spots, Einstein had a photograph.

Of course, you also can call it what you like.

> > The picture that Poincare had, has already been shown by others in this
> > thread to be sufficiently described in his papers.
>
> Sorry, I couldn't read the sketch on this group. Write it or else it is
> futile to continue this discussion.

I already did. Thus it's futile indeed.

[ Mod. note: Please wrap up the discussion, this thread has been going
in circles for quite some time. -ik ]

Harald


0 new messages